You are on page 1of 104

8/30/17

Recommended Books for MRCOG Part 2 Examinations

8/30/17 ELBOHOTY 1 2 ELBOHOTY 8/30/17

—Imagination is more important


than knowledge. Tips before the exam
1. Keep asking Allah to pass the exam, ask your
— Albert Einstein
parents to pray for you, do some good deeds.
—Creativity is especially expressed 2. Sleep enough especially in the coming days, eat
in the ability to make connections fresh fruits, some chocolate, practice light physical
— Graphic designer Tim Hansen
exercises regularly (walking in fresh air)
3. Dont argue about the strange questions alot
4. Answer EMQ & SBA more; conventional study less

3 ELBOHOTY 8/30/17 4 ELBOHOTY 8/30/17

1
8/30/17

Exam tips
1. Answer what your are confident 1st, you can leave query question till
Post exam
you finish, dont leave any question without answering at the end of the
exam. —Intend from now that you will help your
2.You should keep confident, smiling and calm colleagues who will apply for the next
3. Don't discuss questions between the 2 papers or read papers you have
studied a lot. exam
4. Answer the question as you answer for the 1st time don't be happy
that you solve it before. It is usually changed even with change in one
adverb gives another meaning.

5 ELBOHOTY 8/30/17 6 ELBOHOTY 8/30/17

UK law Forms of consent


• UK law • Consent form 1: Patient agreement to
– Act of Parliament – e.g.
• 2005 Mental Capacity Act 2005
investigation or treatment
• 1998 Freedom of information Act • Consent form 2: Parental agreement (or person
• 1967 Abortion Act
• 1990 Human Fertilisation and Embrology Act who has parental responsibility) to investigation
• 2005 Female Genital Mutilation Act or treatment for a child or young person
• 2004 Human Tissue Act
• consent form 4: Form for adults who lack the
– Case law capacity to consent to investigation or treatment
• Lord Fraser competency
• Montgomery

8/30/17 ELBOHOTY 7 8/30/17 ELBOHOTY 8

2
8/30/17

Montgomery ruling Consent in minors


• A doctor must take “reasonable care to ensure that the patient is aware of
any material risks involved in any recommended treatment and of any
reasonable alternative or variant treatment”
• Before the Montgomery ruling in 2015, the
Bolam test was the benchmark for assessing medical negligence.
• The Bolam ruling stated: “A doctor is not guilty of negligence if he has acted
in accordance with a practice accepted as proper by a responsible body of
medical men skilled in that particular art. Putting it another way round, a
doctor is not negligent if he is acting in accordance with such a practice,
merely because there is a body of opinion that takes a contrary view.”

8/30/17 ELBOHOTY 9 8/30/17 ELBOHOTY 10

The Fraser Guideline


• The young person understands the doctors advice Data Protection Act
• The doctor cannot persuade the young person to inform
his/her parents • Covers any personal, identifying information
• The young person is very likely to begin or continue having • Must be kept confidential
sexual intercourse with or without contraceptive treatment
• Must be relevant, accurate and not stored for
• Unless she receives contraceptive advice or treatment the
longer than necessary
young person’s mental or physical health or both are likely
to suffer • Can only be disclosed to the person whose
• The young person’s best interests require the doctor to give information is stored unless exceptional
contraceptive advice or treatment or both without parental circumstances
consent
• Information governance training
8/30/17 ELBOHOTY 11 8/30/17 ELBOHOTY 12

3
8/30/17

Disclosure
Legal requirements
• When the patient gives consent
• Sharing clinically relevant information with other • Notification of disease
staff to assist in the management of a patient.
– Statutory requirements:
• Disclosure without consent may be justified • HIV, gonorrhoea, syphilis, clostridium difficile
where failure to do so may expose the patient or
others to risk or death or serious harm (when a • Notification of deaths to the coroner
patient may be a victim of neglect or abuse) – Neonatal deaths including <24 weeks if signs
• Personal information may be disclosed in the of life, Post op deaths, unknown cause of
public interest death
• E.G. Where a disclosure may assist in the prevention • MBRRACE
or detection of a serious crime, abuse of children. – Continual audit of maternal deaths
8/30/17 ELBOHOTY 13 8/30/17 ELBOHOTY 14

Freedom of Information Act 2000 Freedom of Information


• The FOI allows anyone from anywhere to request any information
from the public domain.
The Act gives right to access information held by public • Exceptions:
bodies including the NHS • Information under DPA 1998
• Information with staff specific details
• Commercially sensitive information such as trade secrets.

• Any personal details such as names in minutes should be blacked


out or only initials used.

• These usually go to a coordinating office at the Trust who will then


decide if the request is valid. The Trust office will then contact the
relevant department if they need assistance and will do so in writing.
• Trust has 20 days to respond to request

8/30/17 ELBOHOTY 15 8/30/17 ELBOHOTY 16

4
8/30/17

Caldicott Principles
Caldicott Guardian
• All NHS organisations and local authorities
which provide social services must have a
Caldicott Guardian
• A senior person should be nominated in each
NHS organisation, including the Department of
Health and associated agencies, to act as a
"guardian". The "guardian" should normally be
a senior health professional or be closely
supported by such a person.
8/30/17 ELBOHOTY 17 8/30/17 ELBOHOTY 18

• Justify the purpose(s)


Every single proposed use or transfer of patient identifiable information
within or from an organisation should be clearly defined and scrutinised, with • Everyone with access to patient identifiable information
continuing uses regularly reviewed, by an appropriate guardian. should be aware of their responsibilities
• Don't use patient identifiable information unless it is necessary Action should be taken to ensure that those handling patient
Patient identifiable information items should not be included unless it is
essential for the specified purpose(s) of that flow. The need for patients to be
identifiable information - both clinical and non-clinical staff - are
identified should be considered at each stage of satisfying the purpose(s). made fully aware of their responsibilities and obligations to
• Use the minimum necessary patient-identifiable information respect patient confidentiality.
Where use of patient identifiable information is considered to be essential, • Understand and comply with the law
the inclusion of each individual item of information should be considered and
Every use of patient identifiable information must be lawful.
justified so that the minimum amount of identifiable information is transferred
or accessible as is necessary for a given function to be carried out. Someone in each organisation handling patient information
• Access to patient identifiable information should be on a strict need-to- should be responsible for ensuring that the organisation
know basis complies with legal requirements.
Only those individuals who need access to patient identifiable information • The duty to share information can be as important as the
should have access to it, and they should only have access to the
duty to protect patient confidentiality
information items that they need to see. This may mean introducing access
controls or splitting information flows where one information flow is used for Professionals should in the patient's interest share information
several purposes. within this framework. Official policies should support them
8/30/17 ELBOHOTY 19
doing so. 8/30/17 ELBOHOTY 20

5
8/30/17

Sex and the law


The Children Act • A child does not become an adult until they
• Duty of disclosure to Child Protection Services reach their 18th birthday
– With agreement if possible
• The age of consent to sex is 16 years old
• The law permits the disclosure of confidential information necessary
to safeguard a young person (practicing sex under age of 13 or the • Sexual activity with under 13-year-olds is
other partner has authority over the girl or big difference in age) . unlawful as they do not have legal capacity to
• Different between fetus and child rights!!
consent to it.
• It therefore constitutes rape
• Any sexual activity involving under 16-year-
olds is unlawful although the law takes into
account the circumstances of peer-to-peer sex
8/30/17 ELBOHOTY 21 8/30/17 ELBOHOTY 22

child sexual exploitation Different forms


• Sexual exploitation of children and young • It can occur through the use of technology without the
people under 18 years of age involves child’s immediate recognition; for example being
exploitative situations, contexts and persuaded to post sexual images on the
internet/mobile phones without immediate payment or
relationships where young people (or a third gain.
person or persons) receive ‘something’ (e.g.
• In all cases, those exploiting the child/young person
food, accommodation, drugs, alcohol, have power over them by virtue of their age, gender,
cigarettes, affection, gifts, money) as a result of intellect, physical strength and/or economic or other
them performing, and/or another or others resources.
performing on them, sexual activities. • Violence, coercion and intimidation are common,
involvement in exploitative relationships being
characterised in the main by the child or young
8/30/17 ELBOHOTY 23
person’s limited
8/30/17
availability of choice resulting from24
ELBOHOTY
their social/economic and/or emotional vulnerability.

6
8/30/17

8/30/17 ELBOHOTY 25 8/30/17 ELBOHOTY 26

• Confidentiality is key to encouraging children to access health


Management care when they need it and so any potential breach of
confidentiality (disclosure of information without the child’s
• CSE is always a safeguarding issue. consent) needs to be formally considered as part of this
process.
• If the clinician has any concerns (even in the • Often a discussion with a senior colleague is the best first step
absence of a clear disclosure from a child), it is in this process.
imperative that they raise these concerns with • If the child is at immediate risk of danger then local
the child and share information by following safeguarding protocols must be followed immediately to
minimise this risk.
local safeguarding protocols. • If the clinician has any concerns about a child’s physical or
• All clinicians working with children should be mental health then onward referral for assessment and
aware of named individuals for safeguarding in treatment may also be required.
• Clear documentation is important, both for the sharing of
their trust and where relevant, the lead midwife information and for evidence should a court case ensue.
for safeguarding should be informed.
8/30/17 ELBOHOTY 27 8/30/17 ELBOHOTY 28

7
8/30/17

• What is the primary duty of the Caldicott guardian?



a. To conduct workshops for patients’ awareness of their
rights
b. To make sure regular clinical audits are happening
15 . When prescribed complication in c. To protect staff interests
OB&GYN as very rare means: d. To protect patient confidentiality issues
e. To provide litigation support to patients who feel unfairly
A.1in 1000 treated

B .1in 1500

C .1in1000-1/10,000

D .<1/10,000
 d

8/30/17 ELBOHOTY 29 8/30/17 ELBOHOTY 30

EMQ
Options
Patient Down syndrome with HMB affecting
A. non malficience
her quality of life accompanied by her
B.beneficience mother who agree to offer MIRENA to her
C. vieracy daughter .You discuss with patient about
D.paternalism MIRENA pros and cons.
E. autonomy
F.justice
G.others
• Which of the principles of the options best
describes the coming situation
8/30/17 ELBOHOTY 31 8/30/17 ELBOHOTY 32

8
8/30/17

• Autonomy • Patient with IUGR with abnormal CTG


need CS patient refused and said she
relies on nature and everything will be
ok.

8/30/17 ELBOHOTY 33 8/30/17 ELBOHOTY 34

• The answer is: E. Autonomy

8/30/17 ELBOHOTY 35 8/30/17 ELBOHOTY 36

9
8/30/17

• 117.Pregnant at 36 wks ask for


induction because her husband will
• The answer is: A. Non malficence
travel somewhere you refuse to offer
her induction

8/30/17 ELBOHOTY 37 8/30/17 ELBOHOTY 38

• B.beneficience

8/30/17 ELBOHOTY 39 8/30/17 ELBOHOTY 40

10
8/30/17

SBA

• B

8/30/17 ELBOHOTY 41 8/30/17 ELBOHOTY 42

• D

8/30/17 ELBOHOTY 43 8/30/17 ELBOHOTY 44

11
8/30/17

• b

8/30/17 ELBOHOTY 45 8/30/17 ELBOHOTY 46

• d

8/30/17 ELBOHOTY 47 8/30/17 ELBOHOTY 48

12
8/30/17

• C

8/30/17 ELBOHOTY 49 8/30/17 ELBOHOTY 50

• E

8/30/17 ELBOHOTY 51 8/30/17 ELBOHOTY 52

13
8/30/17

EMQ: option list EMQ: Case scenarios


A Abortion under clause A • 1. A 22 year old woman requests termination of pregnancy. Her last
B Abortion under clause B period was 8 weeks ago and she had stopped taking the combined
C Abortion under clause C
OCP due to migraines. The pregnancy is unplanned and she
D Abortion under clause D explains that she is due to be a bridesmaid at her sister’s wedding in
E Abortion under clause E 3 month times and is worried that she won’t fit into her dress, hence
F Decline to treat
she requests a termination.
G Decline to treat and arrange second opinion
H Decline to treat due to conscientious objection • Answer [ ]
I Decline to treat due to medical contraindications
J Prescribe levonorgestrel • 2. Your colleague is a Catholic with a very strong faith and
K Prescribe mifepristone discusses with you that he has seen a patient who has requested a
L Prescribe misoprostol termination of pregnancy as she has fallen pregnant due to an extra-
M Prescribe ulipristal marital affair. She requested TOP as she feels that she can’t cope
N Surgical TOP with the pregnancy due to the anxiety that her marriage will fail and
The following clinical scenarios relate to women seeking termination of pregnancy. For she couldn’t cope with being a single parent. He is struggling to
each scenario choose the single most appropriate option from the list above. Each
option may be used once, more than once or not at all. advise the patient.
8/30/17 ELBOHOTY 53
• Answer [ 8/30/17] ELBOHOTY 54

• Mr Jones writes to your hospital asking to see his


wife’s medical record, specifically the labour and
EMQ: Case scenarios delivery record when she had a failed trial of rotational
• 1. A 22 year old woman requests termination of pregnancy. Her last forceps followed by an emergency caesarean section
period was 8 weeks ago and she had stopped taking the combined two years ago. Their son has developmental delay
OCP due to migraines. The pregnancy is unplanned and she
explains that she is due to be a bridesmaid at her sister’s wedding in and Mr Jones suspects intrapartum asphyxia is the
3 month times and is worried that she won’t fit into her dress, hence cause.
she requests a termination.
• Answer [ F – decline to treat ]
• Chose the most appropriate action:
• A Allow him to read the notes
• 2. Your colleague is a Catholic with a very strong faith and
discusses with you that he has seen a patient who has requested a
• B Ask him to make a Freedom of Information request
termination of pregnancy as she has fallen pregnant due to an extra- • C Decline to provide a copy of the notes for him
marital affair. She requested TOP as she feels that she can’t cope
with the pregnancy due to the anxiety that her marriage will fail and • D E mail a copy of the notes to him
she couldn’t cope with being a single parent. He is struggling to
advise the patient. • E Provide a photocopy of the notes
• Answer [ 8/30/17
G – decline to treatELBOHOTY
and arrange second opinion ] 55 8/30/17 ELBOHOTY 56

14
8/30/17

• Mr Jones writes to your hospital asking to see his SBA


wife’s medical record, specifically the labour and
• Mrs TC booked in her second pregnancy, she was sure
delivery record when she had a failed trial of rotational of her dates and her scans at 12 and 20 weeks agreed
forceps followed by an emergency caesarean section with the EDD. At 36/40 she attended ANC with her
two years ago. Their son has developmental delay husband and Mother-in-law insisting that her dates were
and Mr Jones suspects intrapartum asphyxia is the wrong and that she was in fact 40 weeks. She was
cause. requesting induction of labour as she was now
“overdue”. She became very angry and aggressive when
• Chose the most appropriate action: the concordance of scans and dates were pointed out to
• A Allow him to read the notes her Later she spoke to the midwife when her husband
• B Ask him to make a Freedom of Information request and Mother-in-law were out of the room. She admitted
that she and her husband had split temporarily and she
• C Decline to provide a copy of the notes for him had a fling and conceived immediately. She and her
• D E mail a copy of the notes to him husband then re-united and he thought the baby was
his. Her Mother-in-law had just calculated the dates and
• E Provide a photocopy of the notes
was worried she was overdue. She asked the midwife to
8/30/17 ELBOHOTY 57
lie for her 8/30/17
and arrange induction
ELBOHOTY
to save her marriage58

Select the single most appropriate


SBA
action from the list below:
• A Advise husband and mother-in-law that due • Select the single most appropriate action from the
date has changed and she is 40 weeks list below:
• A Advise husband and mother-in-law that due
• B Advise them all that dates are correct and date has changed and she is 40 weeks
she is 36 weeks • B Advise them all that dates are correct and she
• C Advise patient on her own that you can’t is 36 weeks
change the dates • C Advise patient on her own that you can’t
• D Alter medical notes and arrange induction as change the dates
requested • D Alter medical notes and arrange induction as
requested
• E Arrange induction in 2 weeks at “T+14” in • E Arrange induction in 2 weeks at “T+14” (in fact
fact 38 weeks
8/30/17 ELBOHOTY 59 38 weeks) 8/30/17 ELBOHOTY 60

15
8/30/17

EMQ option list


A
B
Carry out MCA assessment
Inform social services
EMQ
C Inform Trust’s lawyer
D Obtain consent from partner/next of kin • 1. Mrs AB: G2 P1, previous C/S for FTP at 9cm.
E Proceed with out consent in the best interests of the baby Booked with independent midwife for home
F Proceed without consent in the best interests of the woman birth. Spontaneous labour at term. Good
G Respect the patient’s wishes progress initially, 8cm 6 hours ago, after 4
hours midwife brought patient to labour ward. Pt
The following scenarios relate to intrapartum care. For each declines ARM and EFM. Intermittent
scenario chose the single most appropriate course of action. Each auscultation showed variable decelerations. VE
option may be used once, more than once or not at all. confirms still 8cm, the patient is adamant she
doesn’t want any intervention and declines C/S.
[ ]
8/30/17 ELBOHOTY 61 8/30/17 ELBOHOTY 62

EMQ EMQ
• 2. Mrs DC is a 34 year old woman having an • 1. Mrs AB: G2 P1, previous C/S for FTP at 9cm. Booked with independent
elective C/S under spinal anaesthetic in her third midwife for home birth. Spontaneous labour at term. Good progress initially,
8cm 6 hours ago, after 4 hours midwife brought patient to labour ward. Pt
pregnancy. She had 2 previous sections and has declines ARM and EFM. Intermittent auscultation showed variable
an anterior placenta praevia. The baby is delivered decelerations. VE confirms still 8cm, the patient is adamant she doesn’t want
safely but the placenta is adherent and there is any intervention and declines C/S. [ G ]
considerable bleeding. She is uncomfortable so a • 2. Mrs DC is a 34 year old woman having an elective C/S under spinal
GA is done. Bleeding continues despite a anaesthetic in her third pregnancy. She had 2 previous sections and has an
hysterectomy and her Hb is 31g/l. She is a anterior placenta praevia. The baby is delivered safely but the placenta is
adherent and there is considerable bleeding. She is uncomfortable so a GA
Jehovah’s Witness and had signed an advance is done. Bleeding continues despite a hysterectomy and her Hb is 31g/l. She
directive stating no blood products. The situation is is a Jehovah’s Witness and had signed an advance directive stating no
becoming critical. [ ] blood products. The situation is becoming critical. [ G ]

8/30/17 ELBOHOTY 63 8/30/17 ELBOHOTY 64

16
8/30/17

For each of the scenarios described choose the single most


Options: appropriate statement concerning the obtaining of patient consent
with respect to the law in England. Each option may be used
once, more than once, or not at all.
a. Inform the police. – A 34 yrs old woman with FGM has delivered
b. Inform child safeguard smoothly, giving birth to a healthy baby girl. Using
c. Inform social services. FGM risk assessment tool, the born baby was
d. Document in theSocial Care Information Centre assessed as being at low risk.
database with ananomysation.
– 34 yrs old with FGM has delivered vaginally, giving
e. Document in theSocial Care Information Centre
database without ananomysation. birth to a healthy baby boy. There was a tear at
f. Agree with the woman for her requested
FGM scar tissue without bleeding. The woman
procedure. requested to re-suture
g. Refuse the woman's request. –e
h. Offer de-infibulation. –g
• 8/30/17 ELBOHOTY 65 8/30/17 ELBOHOTY 66

For each of the scenarios described choose the single most appropriate
statement concerning the obtaining of patient consent with respect to the
law in England. Each option may be used once, more than once, or not at • A couple are referred to the fertility service because the 24-
all. year-old male partner has developed testicular cancer that
A: Abandon operative procedure and reschedule
B: Defer the operative procedure will require orchidectomy followed by chemotherapy.
C: Fraser competence must be demonstrable before obtaining consent They wish to have his semen stored for artificial
D: Obtain legal advice on individuals who withhold consent for treatment
insemination of his wife if he were to become azoospermic
E: Obtain legal advice on interpretation on the Abortion Act 1967 in the future
F: Obtain legal advice on interpretation on the Human Fertilisation and Embryology
Act 1990 and the Code of Practice of Human Fertilisation and Embryology
Authority Act 2009
G: Obtain legal advice on interpretation on the Mental Capacity Act 2005
H: Parental consent must be obtained before proceeding
I: Paternal consent must be obtained before proceeding
J: Perform additional procedure without explicit consent to do so
K: Proceed without consent in order to save the fetus’ life
L: Proceed without consent in order to save the woman’s life written consent must be obtained.
M: Respect the rights of the putative father of the fetus to withhold consent
N: Respect the rights of the unborn fetus and proceed to delivery Royal College of Obstetricians and
O: Respect the rights of the woman to withhold consent for treatment Gynaecologists. Obtaining Valid Consent. Clinical
P: Specific consent is unnecessary Governance Advice 6. London: RCOG; 2015.
Q: Verbal consent alone is acceptable
R: Verbal consent8/30/17
with witness and case note documentation
ELBOHOTY 67 8/30/17 ELBOHOTY 68
S: Written consent must be obtained

17
8/30/17

• A 13-year-old girl requires a hymenotomy to treat her • A 14-year-old girl attends the gynaecology clinic
cryptamenorrhoea requesting a termination of pregnancy. Her last period was
10 weeks ago. She does not want her parents to know

parental consent must be obtained before Fraser competence must be demonstrable


proceeding. before obtaining consent.
8/30/17 ELBOHOTY 69 8/30/17 ELBOHOTY 70

An 18-year-old woman is to be seen by the


• A 19-year-old woman suffering from Down syndrome is
referred to the gynaecology clinic with an abdominal mass consultant gynaecologist who is accompanied
that needs an exploratory laparotomy. by a medical student as a part of his clinical
She is an orphan and lives in a residential care home and attachment
is accompanied to the clinic by one of the home’s
healthcare assistants. She has a low IQ

obtain legal advice on interpretation on the verbal consent alone is acceptable.


Mental Capacity Act 2005.
8/30/17 ELBOHOTY 71 8/30/17 ELBOHOTY 72

18
8/30/17

A 39-year-old woman, P4, has attended the day


case surgical unit for a surgical termination of
pregnancy and insertion of a Mirena® IUS for
which she consented for a week before in the
outpatient clinic. She now appears somewhat
upset and reticent about undergoing the
procedure

defer the operative procedure.


8/30/17 ELBOHOTY 73 8/30/17 ELBOHOTY 74

• A 15 year old girl attends outpatient clinic with her


stepmother, who has not adopted her. Her father, who was
married to the patient’s mother at the time of her birth, could
not attend the appointment. The patient is due to have a
Never events
laparoscopic cystectomy for an 8 cm dermoid cyst in a week. • Never events are defined by the NPSA as “Serious
The consultant has deemed the patient to have capacity and largely preventable patient safety incidents that
has explained the procedure, risks and benefit, and
alternatives to her and her parents on a previous clinic visit. should not occur if available preventative
Both the father and stepmother support the patient’s decision measures have been implemented by health care
to proceed with surgery. providers”.
• What is the most appropriate means of completing the • They include incidents such as:
consent form: • wrong site surgery
a. You are required to invite only the patient’s father back to • Retained foreign object post-procedure
sign • Wrong route administration of medication
b. You are required to invite both parents back to sign • Transfusion or transplantation of ABO-incompatible blood
c. The consent form requires only the stepmother’s signature components or organs
d. The consent form requires only the patient’s signature • Overdose of Insulin due to abbreviations or incorrect device
e. Only the patient’s birth mother can sign the consent form
8/30/17 ELBOHOTY
d 75 8/30/17 ELBOHOTY 76

19
8/30/17

• A group of medical students learn together as a


group using a theoretical case of a gynaecology
patient. They read through the case, discuss
• Options potential themes and create learning objectives
• A The Delphi technique
• B The one-minute preceptor incorporating physiological, psychological,
• C Problem-based learning anatomical, ethical and professional aspects,
• D Schema activation
• E Complex procedural hierarchy
amongst other themes. The role of the tutor is
• F Schema refinement as a facilitator to ensure that the students do
• G Snowballing
• H Lectures
not discuss themes at a tangent to the case.
• J Brainstorming
• Peer coaching
• Which teaching method is being described in each scenario?

8/30/17 ELBOHOTY 77 8/30/17 ELBOHOTY 78

• C Problem-based learning • A group of obstetrics and gynaecology


trainees attend a tutorial by their
consultant on the physiology and
endocrinology of polycystic ovarian
syndrome. The tutor then provides real-life
examples of cases of women affected by
the condition and the group discusses how
to solve their clinical problems.

8/30/17 ELBOHOTY 79 8/30/17 ELBOHOTY 80

20
8/30/17

• F Schema refinement • An obstetric trainee begins to learn how to


perform a caesarean section by assisting his
consultant. The consultant and trainee develop
a rapport over a period of time, leading to the
consultant assisting the trainee to perform their
first caesarean section. Eventually, through
ongoing assessment and feedback, the trainee
performs the caesarean section with another
trainee assisting, supervised by the consultant.
8/30/17 ELBOHOTY 81 8/30/17 ELBOHOTY 82

• E Complex procedural hierarchy • A group of obstetrics and gynaecology trainees


ask their consultant for a tutorial about current
fertility treatments. The tutor wishes to know the
trainees’current knowledge and understanding
of the subject so that she can further their
knowledge with her tutorial and therefore, asks
the trainees questions first to build a discussion
that subsequently influences what she includes
in her tutorial
8/30/17 ELBOHOTY 83 8/30/17 ELBOHOTY 84

21
8/30/17

Lead-in.
• A SpR1 has been asked to carry out an audit and 50 sets of
case-notes were given to him to go through them and extract
• Snowballing the necessary data on a Friday afternoon.
• He decided to take the notes home to extract the data.
• On the way home he stoped at his favourite supermarket.
• When he emergesd, his car has been stolen with the notes
inside.
• He reported the theft to the police.

8/30/17 ELBOHOTY 85 8/30/17 ELBOHOTY 86

A Report events to the Caldicott Guardian


Options Question 1.
B Report events to the Chief Executive
C Report events to the General Medical Council • The SPR1 informs you (the Clinical Director),
D
E
Report events to the NHSLA as a “never event”
Report events to the NHSLA as a “serious incident”
on the Monday when he returns to work.
F
G
Report events to the NPHSLA as a “never event”
Report events to the NPSLA as a “serious incident” • What action will you take?
H Report events to the Risk Management Team
I Report events to the Root Cause Analysis Team
J Report events to the Trust Information Management Committee
K Suspend the doctor until a full investigation has been done
L The doctor should be informed of the Trust policy about removing notes from the premises.
M Discuss with the legal team, inform the patient, discuss the implications and keep her fully-up-to-date
N Ask the Caldicott Guardian to deal with it

O Ask the Chief Executive to deal with it

P Ask the hospital’s legal team to deal with it

Q Ask the patient’s GP to deal with it

S Tell all those who know about the incident to discuss it with no one else, particularly the patient
8/30/17 ELBOHOTY 87 8/30/17 ELBOHOTY 88

22
8/30/17

• Question 2. • Question 3.
• What action will you take to deal with the • What action will you take in relation to the
SpR? patient whose notes are missing?

8/30/17 ELBOHOTY 89 8/30/17 ELBOHOTY 90

• The first thing is to establish the facts. It is an adverse event and could be
Question 1. serious, so it should go to the Risk Management team as an adverse event.
They will seek out the facts and do a root-cause analysis.
• It will probably turn out that the doctor did not know that there was a policy
• The SPR1 informs you (the Clinical Director), about not removing case-notes from the Trust.
on the Monday when he returns to work. • There is also the wider issue of education on the subject, which might be
appropriate for the induction process for new staff.
• What action will you take? • The notes may contain little information that would amount to a serious
breach of confidentiality – the patient might only have attended once and
• H. Report events to the Risk Management that to the A&E department with a minor injury. And absence of these notes
Team would have no adverse effect on any future treatment. But the opposite
could be true with the potential for leakage of highly confidential information.
• Important information may no be longer available if the woman needs
treatment - someone will have to use their imagination to find as much as
they can – e.g. from hospital computer records, GP records, lab or radiology
reports stored on departmental computers.

8/30/17 ELBOHOTY 91 8/30/17 ELBOHOTY 92

23
8/30/17

• The doctor should be informed of the Trust policy about


removing notes from the premises. It would not be a major
disciplinary issue unless the doctor was known for his disregard
for Trust policies or other disciplinary issues and I don’t think it
would need to involve the Postgraduate Dean.
• Question 2.
• Terms and Conditions relating to employment contracts for
• What action will you take to deal with the junior doctors which states: “….allegations of misconduct
SpR? against a doctor in training should be considered initially as a
training issue and the postgraduate dean and educational
• L. The doctor should be informed of the supervisor should be involved in dealing with these issues.
Trust policy about removing notes from the Every employer will have their own procedure to follow (based
on national guidance)…… for junior doctors, all matters should
premises. first be considered as a training matter”.

8/30/17 ELBOHOTY 93 8/30/17 ELBOHOTY 94

• We have a responsibility to inform patients of


• Question 3. matters that might affect them. I would also talk
• What action will you take in relation to the to the legal team as there could be legal
patient whose notes are missing? consequences for the Trust – the woman might
• M. Discuss with the legal team, inform the sue. But she needs to know that the information
patient, discuss the implications and keep her in the notes is missing. She will need to provide
fully-up-to-date as much information as she can about her
medical history if she is seen again as a patient
before the notes are recovered. And there
might be very embarrassing stuff which could
8/30/17 ELBOHOTY 95
lead to blackmail
8/30/17
or other
ELBOHOTY
problems. 96

24
8/30/17

A 19 year old woman is admitted to intensive care with a sever asthma attack after being
prescribed ulipristal acetate for emergency contraception. She was known to be a poorly
controlled asthmatic. The ST1 admits to you they didn’t check the contraindications for
this drug before prescribing it.

What is the most appropriate next step?


Options:
Options:

a) Arrange a tutorial for all the ST1s and ST2s a) Arrange a tutorial for all the ST1s and ST2s
b) Submit an incident report b) Submit an incident report
c) Debrief the ST1 with a written record
d) Inform the Royal College Tutor
c) Debrief the ST1 with a written record
e) Inform the Medical Director d) Inform the Royal College Tutor
e) Inform the Medical Director

8/30/17 ELBOHOTY 97 8/30/17 ELBOHOTY 98

• Options: • A woman was diagnosed with Ca cx. It


• A. Route cause analysis is found that she had a previous smear
• B. Incident form result showing severe dyskariosis but
she moved house & was not called
• C. Never even back for further management.
• D. Inform GMC
• E. SIRI
• F. Inform police
• G. Inform hospital legal department
8/30/17 ELBOHOTY 99 8/30/17 ELBOHOTY 100
• H. Audit

25
8/30/17

• A woman has an intrapartum stillbirth. Despite


extensive discussion and explanation of the
management of the pregnancy and delivery
with her consultant, she still expresses
dissatisfaction. She indicates that she wishes to • The correct answer is the Patient Advice
explore further whether the stillbirth should and Liaison Service. If a patient has a
have been avoided. On a ward round she asks complaint or concern, it is best dealt with
you whom she should contact for help. To by the provider of the health care in the
which of the following organisations would you first instance. Other organisations may be
direct her in the first instance? appropriate if the initial response is not
• Care Quality Commission satisfactory.
Clinical Commissioning Group
General Medical Council
Patient Advice and Liaison Service
Patient Association
8/30/17 ELBOHOTY 101 8/30/17 ELBOHOTY 102

• Clinical governance is a framework through which


NHS organisations are accountable for continuously
improving the quality of their services and safe
guarding high standards of care by creating an
• Clinical effectiveness
environment in which excellence in clinical care will
flourish. Which aspects of clinical governance are
characterised by the following definitions:
Ensuring that things are done in the right way, by the
right person, in the right place, with the right result.
• Clinical audit
• Clinical effectiveness
• Patient involvement
• Risk management
• Strategic leadership
8/30/17 ELBOHOTY 103 8/30/17 ELBOHOTY 104

26
8/30/17

• Which aspect of clinical governance is


characterised by the following definition:
Ensuring that health records are accurate • Information management
and up-to-date, that confidentiality is
maintained in respect of patient data and that
all available sources of data are mined to
produce intelligent information for improving
quality.

• Clinical effectiveness
• Information management
• Patient and public involvement
• Risk management
• Strategic8/30/17
leadership ELBOHOTY 105 8/30/17 ELBOHOTY 106

• Which aspect of clinical governance is characterised by


• Clinical audit
the following definition: A quality improvement process
that seeks to improve patient care and outcomes through
systematic review of care against explicit criteria and the
implementation of change.

• Clinical audit
• Clinical effectiveness
• Guideline development
• Information management
• Risk management

8/30/17 ELBOHOTY 107 8/30/17 ELBOHOTY 108

27
8/30/17

• A woman in your ward has required blood transfusion. Shortly after


starting transfusion. you've noticed that the bag is labelled with a
different name. You stopped the transfusion & the patient is stable.
• What is the most appropriate next step?
a. Reassure the staff that no complications has occurred& no further
action needed. • A lady suffering from endometrial cancer had undergone
a hysterectomy as part of her treatment. She was
b. Interview the whole staff to know who is responsible for the mixing. unhappy about the care she received on the ward
c. Report what happened in your portifollio. following the procedure and has mentioned this to you a
d. Document the case & complete an incident report. few times on your daily ward rounds.
What would you do in this situation?
e. Resign from the hospital
• Encourage positive thoughts in the lady by discussing better
aspects of care
• Encourage the lady to put in a complaint
• Fill in an incident form regarding the above
d • Provide details of the Patient Advice and Liaison Service
(PALS)
• Reassure the lady that you will talk to the staff involved

8/30/17 ELBOHOTY 109 8/30/17 ELBOHOTY 110

• Every year a hospital accepts 12 new trainee


as a part of the training program. What is
• The answer is provide details of PALS. The the best method to evaluate the new trainee
PALS is available in all hospitals and offers before they engage into the hospital?
confidential advice, support and information a. Applying ?guidelines from previous patch [or
on health-related matters to patients, their
families and their carers. They would be able something like that].
to help in the above situation and address the b. Written exam.
lady’s concerns effectively. Although c. Induction interview.
reassuring the lady and alerting staff to the
concerns is important, patients may feel that d. Directly observed skills.
they are not being taken seriously by just e. MRCOG exam
talking and reassurance.
c

8/30/17 ELBOHOTY 111 8/30/17 ELBOHOTY 112

28
8/30/17

• The RCOG exam has two componenets written • The goal of formative assessment is to monitor
and OSCE, Which choice best describes each: student learning to provide ongoing feedback that can
be used by instructors to improve their teaching and
• by students to improve their learning. Summative
• Written OSCE assessment
• Formative Summative • The goal of summative assessment is to evaluate
student learning at the end of an instructional unit by
• Formative Formative comparing it against some standard or benchmark.
• Summative Formative
• Summative Summative
8/30/17 ELBOHOTY 113 8/30/17 ELBOHOTY 114

• In your hospital it was decided to use FFN for all • When couple is married at the time of still
patients who present with risk of preterm labour. After birth of their child,legal duty of registration
6 months only you realized that the stock that was lies on whom
ordered has all finished. You want to know whether
you have underestimated the number of patients that • Mother
present to your hospital or what. What is your • Health care professional
management:
• Father
• -Clinical effectiveness
• Couple as a unit
• -Arrange meeting with unit managers –
• Run an audit • Anyone can register as per situation
• -Make a research
8/30/17 ELBOHOTY 115 8/30/17 ELBOHOTY 116

29
8/30/17

Study methodology and


statistics
• Health care professional • What do you need to know?
– Types of studies
– Types of Data
– Tests of significance and CI
– Errors
– Screening
– Systematic reviews and metaanalysis

8/30/17 ELBOHOTY 117 8/30/17 ELBOHOTY 118

• you designed a questionare to detect the *study


prevelance of dysmenorrhea among high A.cohort
school girls
B.case control
• What study design best describe this study
• A- cohort study
C.retrospective observational study
• B- case controlled study D.systematic review
• C- case series E. metaanalysis
• D- randomized controlled study F.RCT
• E- cross sectional study G.other option
8/30/17 ELBOHOTY 119 8/30/17 ELBOHOTY 120

30
8/30/17

• 118. DR conducted study over 15 yrs to • The answer is: A. Cohort


see effect of Carboplatin on 5yrs
survival of patient with cancer.

8/30/17 ELBOHOTY 121 8/30/17 ELBOHOTY 122

• 119. DR looks in literature to see effect • The answer is: D. Systematic review
of MIRENA in HMB.

8/30/17 ELBOHOTY 123 8/30/17 ELBOHOTY 124

31
8/30/17

• 120. DR conduct study among doctors • The answer is: B. Case control
to see effect of smoking and non
smoking in lung cancer.

8/30/17 ELBOHOTY 125 8/30/17 ELBOHOTY 126

Some bored post-membership registrars recently conducted a


study to determine how well abdominal palpation performed as
a screening test for breech presentation.After palpating a woman’s
abdomen she was then scanned to determine the presentation. You wish to investigate the time it takes to perform a
Palpates Palpate total caesarean section on women with and without pre-
breech s A. 6% eclampsia. At the end of the study you have two
cephali groups of women with the duration of caesarean
c B. 8% sections recorded for each. It can't be assumed
Scans show breech 24 8 32 that these are normally distributed.
Scans show 48 320 368 C. 18% Which non-parametric statistic test would you use?
cephalic
D. 33% A. Chi-squared test
Total 72 328 400
E. 50% B. Linear regression analysis
For each of the following questions ,select the best estimate from the C. Mann-Whitney U test
list of options .Each option may be used once more than once or not D. Pearson’s R test
at all.
Ø31. What is the positive predictive value of abdominal palpation for E. Student’s t-test
breech presentation?
Answer : D 24/72 %= 33%
8/30/17 ELBOHOTY 127 8/30/17 ELBOHOTY 128

32
8/30/17

In a sample of 100 women the age at the


delivery of their first child showed a
normal distribution. The mean age at
delivery of their first child was 30 years • Pain is scored from 0 to 10 via visual
old, with a standard deviation of 5 years.
What is the 95% confidence interval? analogue scale. What kind of data is this?
A. 20 – 30 years •
B. 20 – 40 years – Categorical.
– Ordinal.
C. 25 – 35 years
– Nominal.
D. 29 – 31 years
– Interval.
E. 15 – 45 years
– Continuous.
D
8/30/17 ELBOHOTY 129 8/30/17 ELBOHOTY 130

• An oncologist suspects that many women diagnosed as halting


breast cancer also have high levels of work-related stress. What
type of design would be best to test this possible association?
• A description of a study was given
where people were assigned to those
• A Group comparative, randomised controlled trial
with the disease & those without &
• B Crossover, randomised crossover trial followed retrospectively.
• C Prospective cohort study – Cohort.
• D Retrospective case control study – Case control.
• E Prospective observational study – RCT.
d – Cross section.
– Crossover
8/30/17 ELBOHOTY 131 8/30/17 ELBOHOTY 132

33
8/30/17

EMQ. Interpretation.
Which one of the following statements best
Options: describes a type I statistical (α error)?
A. Convincing evidence of benefit.
B. Convincing evidence of harm. A. Rejecting a null hypothesis when there is a
C. No benefit & no harm. difference
D. A trend for benefit with concern of harm.
E. Unethical study. Should be terminated. B. Rejecting a null hypothesis when there is no
F. Underpowered. Repeat study with n=10000.
G. Underpowered. Repeat study with n=20000. difference
H. Odds ratio can't be counted upon
I. Invalid results C. Rejecting the alternative hypothesis when there is
Lead in: A RCT study was carried to evaluate a new drug for the treatment of hypertension a difference
in pregnancy. 2 groups; study group of 1000 women on the new drug & a control group of
1000 women on methyl dopa. Both groups were matched for age & BMI. D. Accepting a null hypothesis when there is a
96. In a preliminary analysis in a 50 women in the study group, the BP of 23 women has
dropped but 13 of them developed drug induced hepatitis. difference
97. At final analysis, BP readings were not available for 40% of the control group & 25%
of the study group. Besides, despite being matched for age, the mean age for the 2
E.
B Accepting the alternative hypothesis when there is
groups came different [28 yrs for control & 34 for study group]. (P value was 0.01) a difference
H8/30/17 ELBOHOTY 133 8/30/17 ELBOHOTY 134
I

90. A study was carried out to compare between the outcome of breech [CS Vs
vaginal]. The women were randomized into planned CS arm & planned VD
arm.

Planned CS arm Planned VD


• 89. Pain is scored from 0 to 10 via visual
Test: Chi squared
n= 1400 n= 1350
*All delivered by CS *1000 delivered vaginally

analogue scale. What kind of data is this? Severe neonatal morbidity: 0.4%
*350 delivered by CS
Severe neonatal morbidity: 1.6% P= 0.002

• A. Categorical. It was concluded that CS is better for breech babies.


What is against this conclusion?
• B. Ordinal.
A. The study was not carried for the intention to treat.
• C. Nominal. B. The groups are not related/comparable.

• D. Interval. C. The results are not statistically significant.


D. The results are not correctly interpreted.
• E. Continuous. E. Wrong test was used.

8/30/17 ELBOHOTY 135


E.8/30/17
Fisher exact test ELBOHOTY 136

34
8/30/17

• A chemotherapy regimen used in patients with ovarian National Confidential Inquiries


carcinoma is tested in a small clinical trial. Out of 50 patients
treated with the new regimen, 25 survive 5 years without • The Confidential enquiries in to Maternal
relapse. Out of 100 patients treated with the conventional
regimen, 25 survive 5 years without relapse. How many Deaths audit has been the longest running
patients need to be treated with the new regimen as opposed in the UK. It is now conducted by an
to the conventional regimen in order for one more patient to independent body and along with the other
survive 5 years without relapse?
A. 2 national enquiry in to peri-operative deaths
B. 4 acts as a useful indicator of the
C. 6 performance of a unit.
D. 8
E. 10

b
8/30/17 ELBOHOTY 137 8/30/17 ELBOHOTY 138

Maternal
Mortality in the UK
2009-12
2006-08 2011-13 2012-14
1952-54

9 per 100,000 8.5 per


11 per 100,000 10 per 100,000 maternities 100,000
90 per 100,000
maternities maternities maternities
maternities

8/30/17 ELBOHOTY 139 8/30/17 ELBOHOTY 140

35
8/30/17

• The stillbirth rate 4.5 per 1,000 total births .


trends –

Unexplained 46%
Placental problems 22%
• Overall there was no statistically significant decrease in – Congenital malformation 6%
the maternal death rate in the UK between 2009-11 and – Intrapartum complications 6%
2012-14.
• Maternal deaths from direct causes remain unchanged • The neonatal mortality rate 2.5 per 1,000 live births.
with no significant change in the rates between 2009-11 – Complications after birth 31%
– Congenital malformation 28%
and 2012-14.
– Prematurity 13%
• The rate of indirect deaths remains high with no – Infections
significant change since 2003, except a decrease in – AN complications
deaths due to influenza. – Intrapartum complications

• Maternal suicides have now been reclassified by the • The perinatal mortality rate 6 per 1,000 total births
– Unexplained
World Health Organisation as a direct cause of maternal
– Congenital malformation
death. The rate of maternal death by suicide remains – Prematurity
unchanged since 2003 and maternal suicides are now
• The extended perinatal mortality rate 7 per 1,000 total
the leading cause of direct maternal deaths occurring births
8/30/17 ELBOHOTY 141 8/30/17 ELBOHOTY 142
within a year after the end of pregnancy.

• The World Health Organizationdefines perinatal


mortality as the "number of stillbirths and
deaths in the first week of life per 1,000 total
births, the perinatal period commences at 22
completed weeks of gestation and ends seven
completed days after birth

8/30/17 ELBOHOTY 143 8/30/17 ELBOHOTY 144

36
8/30/17

* maternal mortality
A.direct
B.indirect
C.coincidental
D.accidental
E.late
F.not maternal death

8/30/17 ELBOHOTY 145 8/30/17 ELBOHOTY 146

• 80 . Lady with pre-eclampsia developed


ICH spent 5wks in ICU and died • The answer is: A. Direct
eventually.

8/30/17 ELBOHOTY 147 8/30/17 ELBOHOTY 148

37
8/30/17

• 81 . . Lady is collapsed 48 hrs post • The answer is: B. Indirect


delivery postmortem was Esimenger
synd

8/30/17 ELBOHOTY 149 8/30/17 ELBOHOTY 150

• Lady was murdered by her husband 4 • The answer is: C Coincidental


wks postpartum.

8/30/17 ELBOHOTY 151 8/30/17 ELBOHOTY 152

38
8/30/17

• A 28-year-old woman dies at 47 days


• What are the three leading three direct causes
postpartum following aspiration during an
of maternal death in the UK from the 2012-2014 epileptic seizure. She had a 10 year history of
Confidential Enquiry into Maternal and Child
epilepsy. What is the classification of this
Health (CEMACH) report?
maternal death?
• A. Infection, pre-eclampsia and VTE
• Early direct maternal death
• B. Pre-eclampsia, haemorrhage and infection
• Early indirect maternal death
• C. Thrombosis, sepsis, haemorrhage
• Late coincidental maternal death
• D. Thrombosis, pre-eclampsia and amniotic • Late direct maternal death
fluidc embolism
• Late indirect maternal death
• E. Infection, pre-eclampsia and amniotic fluid
embolism
8/30/17 ELBOHOTY 153 8/30/17 ELBOHOTY 154

• The correct answer is late indirect


maternal death. A maternal death that
occurs 6 weeks following child birth is
termed as late maternal death. If death
occurs of a pre-existing medical condition
it is called an indirect maternal death.

8/30/17 ELBOHOTY 155 8/30/17 ELBOHOTY 156

39
8/30/17

Effects of
Surgery?

Surgical considerations

8/30/17 ELBOHOTY 157 8/30/17 ELBOHOTY 158


158•
Fit for Surgery? Mythen MG. Anesthesia and Analgesia: April and May 2011

Enhanced Recovery pathway


• Optimising pre operative • Planned mobilisation
haemoglobin levels • Rapid hydration &
• Managing pre existing co nourishment
Referral from morbidities e.g. diabetes • Short time
• Appropriate IV therapy
Primary Care • Alternatives to surgeries • Optimised Fluid
• No wound drains
• Minimal invasive surgery Hydration
• No NG (bowel surgery)
• Reduced starvation
• Catheters removed
• No / reduced bowel
Pre- preparation ( bowel
early
Operative • Regular oral analgesia
surgery)
• Paracetamol and
NSAIDS
• Avoidance of opiate-
Admission based analgesia where
possible or
• Optimised health / medical administered topically
condition
• Informed decision making Intra-
• Pre operative health & risk
Operative
assessment - CPEX
• PT information and • Minimally invasive • Audit &
expectation managed surgery outcome
• Admission planning • Use of transverse Post- measures
• Screening of infections incisions Operative
• No NG tube (bowel
surgery)
• Use of LA with sedation • Dischare on planned day
• Epidural management • Therapy support (stoma, Follow
(inc thoracic) physio) Up
8/30/17 • OptimisedELBOHOTY
fluid • 24hr telephone follow up 159 8/30/17 ELBOHOTY 160
management 159

40
8/30/17

8/30/17 ELBOHOTY 161 8/30/17 ELBOHOTY 162

8/30/17 ELBOHOTY 163 8/30/17 ELBOHOTY 164

41
8/30/17

Principles of closure
- closure of the fascia should be 1.5 cm
from the edge with the suture 1 cm
apart.
-interrupted sutures are considered with
evidence of infection
-Excessive tension on sutures leads to
tissue necrosis and eventual failure of
closure
8/30/17 ELBOHOTY 165 8/30/17 ELBOHOTY 166

Suture & needle types Suture pack labelling

8/30/17 ELBOHOTY 167 8/30/17 ELBOHOTY 168

42
8/30/17

Needles

8/30/17 ELBOHOTY 169 8/30/17 ELBOHOTY 170

Instruments: Needle (I)


• The main types of needle include:
– Tapered
• Gradually taper to the point and cross-section
reveals a round, smooth shaft
• Used for tissue that is easy to penetrate, such as
bowel or blood vessels

– Cutting
• Triangular tip with the apex forming a cutting
surface
• Used for tough tissue, such as skin (use of a
tapered needle with skin causes excess trauma
because of difficulty in penetration)

– Reverse cutting needle


• Similar to a conventional cutting needle except
the cutting edge faces down instead of up
• This may decrease the likelihood of sutures
8/30/17 ELBOHOTY 171 8/30/17 pulling through soft tissue
ELBOHOTY 172

43
8/30/17

8/30/17 ELBOHOTY 173 8/30/17 ELBOHOTY 174

Absorbable Nonabsorbabl
sutures e sutures

Natural Natural
Synthetic sutures Synthetic sutures

Chromic gut Silk


50% tensile Unfortunately,
strength by 10 this is the most
Polyfilament Polyfilament
days reactive of
s Monofilaments s Monofilaments nonabsorbable
sutures.
Polyglycan 910 (Vicryl)
Polyglycolic acid (Dexon) Polyglyconate (Maxon)
Polydioxanone (PDS) • Dacron-polyester (Mersilene) Polypropylene (Prolene)
50% tensile strength for
25 days 50% tensile strength for at low tissue reactivity and
least 40 days great tensile strength but
Poliglecaprone 25 (Monocryl) low knot security due to
high memory
8/30/17 ELBOHOTY 175 8/30/17 ELBOHOTY 176

44
8/30/17

8/30/17 ELBOHOTY 177 8/30/17 ELBOHOTY 178

Procedure` Suggested Suture Material

Episiotomy Continous, Vicryl rapid No. 0 or 2-0; taper point needle, 35-mm
Anal mucosa Vicryl No. 3-0; Round-body taper-point needle, 26-mm

Continuous or interrupted PDS or Vicryl No. 2-0 or 3-0; Round-body taper-point needle, 26-mm
Anal sphincter

Uterine incision Continuous 2 layers; Vicryl No. 1; Round-body taper-point needle. 50-mm
• Transverse incision:
Vicryl or PDS No. 1 or 2; taper cut needle or reversed-cutting needle
Rectus sheath
• Longitudinal incision: (mass closure is recommended)
PDS or Prolene No. 1 or 2; taper cut needle or reversed-cutting needle
Peritoneum Recommended not to be closed (neither parietal or visceral)

Fat & subcutaneous Vicryl No. 2-0; Round-body taper-point needle

B-Lynch haemostatic suture Monocryl No. 1; Round-body taper-point needle; size = 70 mm.

Urinary bladder repair Vicryl No. 2-0 or 3-0; Round-body taper-point needle
Ureteric repair Vicryl No. 3-0; Round-body taper-point needle
Small intestine repair Vicryl No. 3-0; Round-body taper-point needle
Prolene No. 2-0; reverse-cutting needle, or
Skin Closure
Vicryl rapid No. 2-0; reverse-cutting needle

Ovarian pedicle Vicryl No. 0 or 1

Uterine pedicel Vicryl No. 0 or 1

Cardinal ligament Vicryl No. 0 or 1

8/30/17 ELBOHOTY 179 Uterosacral ligament 8/30/17 Vicryl No. 0 or 1 ELBOHOTY 180
Vaginal vault Vicryl No. 0 or 1 taper cut needle or reverse-cutting needle

45
8/30/17

Properties of Electricity

• VOLTAGE =Force pushing current through the resistance, measured


in volts
• CURRENT =Flow of electrons during a period of time, measured in
amperes
• CIRCUIT =Pathway for the uninterrupted flow of electrons
8/30/17 ELBOHOTY 181 • RESISTANCE
8/30/17 = Obstacle to the flow of current, measured in ohms
ELBOHOTY 182
(impedance = resistance)

Electrosurgical unit (ESU)

• Direct current would be transmitted


through body tissue at 60 cycles per
second (Hz), excessive neuromuscular
stimulation and perhaps electrocution
would result.
• The ESU is a high frequency alternating current generator.
8/30/17 ELBOHOTY 183 8/30/17 ELBOHOTY 184
• Most have their power output displayed in Watts

46
8/30/17

Alternate current
• Characterized high frequency (400 kHz to 3.3 Components of electrosurgery
MHz)
• At this frequency electrosurgical energy can • Current and voltage
pass through the patient with no risk of provided by generator
electrocution. • Circuit: passes
through patient
• Tissue provides
resistance and
produces heat
8/30/17 ELBOHOTY 185 8/30/17 ELBOHOTY 186

Types of electrosurgery

8/30/17 ELBOHOTY 187 8/30/17 ELBOHOTY 188

47
8/30/17

Effects of modified waveforms in tissue

Complication Cause
• Superficial burns from pooling of inflammable liquid, such as spirit-based skin
preparations.
• Wrongly placed patient plate electrode.
Burns
• Retained heat in the electrode touching skin.
• Poorly insulated diathermy lead.
• Inadvertent use.
Electrocution • Poorly insulated diathermy leads.

• Can reprogram pacemaker.


• Pacemaker wires may conduct current to the heart causing myocardial
Pace-maker
burns.
interference
Use preferentially bipolar, use monopolar only with short, low power waves, with
return electrode placed well & away from the pace maker site.

Coupling • Conduct of current in other metal conductors, such as laparoscopy ports.

8/30/17 ELBOHOTY 189


• 8/30/17
A capacitor is formedELBOHOTY
when two conducting surfaces are separated
190
by an
Capacitation
insulator, such as the mixed material laparoscopic port.

Diathermy in Gynaecology-
General Use
•Benign Cervical Lesions-
•CIN (LEETZ / LEEP)-
•Ovarian Drilling in PCOD-
•Endometriosis-
•Hysteroscopic surgery

8/30/17 ELBOHOTY 191 8/30/17 ELBOHOTY 192

48
8/30/17

Media used in hysteroscopy

Medium Advantages Disadvantages


CO2 • Well-tolerated, convenient and • Most serious disadvantage is that of
easy to use gas embolism
• Visibility can be distorted in the
presence of bleeding
• Shoulder tip pain
Normal • Low cost and readily available • Used only for operative
saline • Reduce vasovagal episodes hysteroscopy in bipolar
electrosurgical operative
hysteroscopy
1.5% • Electrolyte-free and non- • Excessive absorption causes
glycine conductive hyponatremia.
• Used in monopolar electrosurgical • Severe overload can lead to
operative hysteroscopy haemolysis, coma and death

8/30/17 ELBOHOTY 193


no significant difference between the pain experienced with the use of carbon
ELBOHOTY 8/30/17
194
dioxide versus normal saline for outpatient hysteroscopy

lithotomy
Surgical positions

Lithotomy position is defined as supine position of the body with the legs separated, flexed
and supported in raised stirrups.

195 ELBOHOTY 8/30/17 196 ELBOHOTY 8/30/17

49
8/30/17

Trendelenburg (45 degree) Lloyd Davis (leg flexed 15 & head down 30)

197 ELBOHOTY 8/30/17 198 ELBOHOTY 8/30/17

Recommendations for minimising


Laparoscopy inadvertent bowel injury during closed entry
laparoscopy:
• Ensure that the operating table is flat.
• Use a vertical incision at the base of the umbilicus.
• Insert the Veress needle only to the depth required to
penetrate the peritoneum.
• Use confirmatory tests of intra peritoneal placement;
for example, Palmer’s test or low insufflation pressure.
• Inflate to 20–25 mmHg before inserting the primary
trocar.
• Perform a 360º inspection following insertion of the
laparoscope.
• Carry out careful inspection on removing the
8/30/17 ELBOHOTY 199
laparoscope
8/30/17
at the end ELBOHOTY
of the operation. 200

50
8/30/17

Laparoscopic entry
techniques
• The most effective way to, reduce complications of
laparoscopic entry is to optimise insertion of the
Intra umbilical
primary trocar and cannula.
entry Veress
needle (closed)
laparoscopic entry
• Alternative entry site (palmers point)
technique
8/30/17 ELBOHOTY 201 8/30/17 ELBOHOTY 202

Verres needle
• It is used to inflate air to the peritoneal cavity
(pneumoperitoneum) through the umbilicus where there is the
thinnest abdominal wall.
• The Veress needle should be sharp, with a good and tested spring
action. A disposable needle is recommended, as it will fulfill these
criteria.

8/30/17 ELBOHOTY 203 8/30/17 ELBOHOTY 204

51
8/30/17

• The operating table should be horizontal at


the start of the procedure.
• The abdomen should be palpated to check
Insertion of Veress needle for any masses and for the position of the
Pencil grip
aorta before insertion of the Veress needle.
Vertical, then towards pelvis
Double “click”

8/30/17 ELBOHOTY 205 8/30/17 ELBOHOTY 206

• The primary incision for laparoscopy : vertical from the


base of the umbilicus. Saline test (palmers test)
• The Veress needle can be inserted at right angles to the
skin 1. Withdraw (ensure no fluid, blood or
• It should be pushed in just sufficiently to penetrate the
fascia and the peritoneum. faecal content)
• Two audible clicks are usually heard as these layers are 2. Inject saline
penetrated.
• Excessive lateral movement of the needle should be 3. Withdraw again if fluid is obtained, this
avoided suggests that the tip of the needle is
either located between the peritoneum
and rectus sheath or placed in an
intrabdominal structure such as
8/30/17 ELBOHOTY 207
omentum, bowel or an adhesion.
8/30/17 ELBOHOTY 208

52
8/30/17

Insuflator:
– Used to insufflate through the verres needle.
– Check gas entering at low pressure (<8mmHg)
– Set pressure cut off to at least 20-25mmHg
– Start at low flow (1L/min)
– After 0.5L flow rate can be increased
– An intra-abdominal pressure of 20–25 mmHg
should be used for gas insufflation before inserting
the primary trocar.
– The distension pressure should be reduced to 12–
15 mmHg once the insertion of the trocars is
complete and trendlenberg position is taken.

8/30/17 ELBOHOTY 209 8/30/17 ELBOHOTY 210

qThe primary trocar should be inserted in a controlled


manner at 90 degrees to the skin, through the incision
at the thinnest part of the abdominal wall, in the base
of the umbilicus.
qInsertion should be stopped immediately the trocar is
inside the abdominal cavity.

8/30/17 ELBOHOTY 211 8/30/17 ELBOHOTY 212

53
8/30/17

2. Primary port closed entry


The greater the gas bubble & abdominal wall tension the less the risk
of bowel injury

Type 2 injury
Type 1 injury (low pressure

Alternative entry

8/30/17 ELBOHOTY 213 8/30/17 ELBOHOTY 214

qHasson open laparoscopic entry


• confirmation that the peritoneum has
been opened should be made by
visualising bowel or omentum before
inserting the blunt tipped cannula.

8/30/17 ELBOHOTY 215 8/30/17 ELBOHOTY 216

54
8/30/17

qPalmer’s point is the preferred


alternative trocar insertion site, except
in cases of previous surgery in this
area or splenomegaly.

8/30/17 ELBOHOTY 217 8/30/17 ELBOHOTY 218

qThey must be inserted under direct vision


perpendicular to the skin, while maintaining the
pneumoperitoneum at 20–25 mmHg.
Secondary ports qThe inferior epigastric vessels should be
visualised laparoscopically to ensure the entry
point is away from the vessels.

8/30/17 ELBOHOTY 219 8/30/17 ELBOHOTY 220

55
8/30/17

8/30/17 ELBOHOTY 221 8/30/17 ELBOHOTY 222

Wound closure:
qOnce the tip of the trocar has pierced Avoid hernia risk by closing rectus sheath:
the peritoneum it should be angled - Midline port sites > 10 mm

towards the anterior pelvis under - Lateral port sites > 7 mm

careful visual control until the sharp tip


has been removed.
qSecondary ports must be removed
under direct vision to ensure.

8/30/17 ELBOHOTY 223 8/30/17 ELBOHOTY 224

56
8/30/17

serious complications
Major laparoscopic
q The overall risk of serious complications from diagnostic
laparoscopy, approximately 2-3 women in every 1 000: complications
– damage to bowel
– Bladder
– uterus
– major blood vessels
Urinary

q These would require immediate repair by laparoscopy or


laparotomy (uncommon).
Uterus
q However, up to 15% of bowel injuries might not be
diagnosed at the time of laparoscopy
Bowel Vascular
8/30/17 ELBOHOTY 225

qSurgeons must be aware of the Serious risks


• The overall risk of serious complications from
increased risks in women who are diagnostic laparoscopy is approximately 2 in 1000
•obese women (uncommon). This includes damage to the
bowel, bladder, ureters, uterus or major blood vessels
•significantly underweight which would require immediate repair by laparoscopy
•previous midline abdominal or laparotomy (open surgery is uncommon). However,
incisions up to 15% of bowel injuries might not be diagnosed at
the time of laparoscopy.
•peritonitis or inflammatory bowel • Failure to gain entry to the abdominal cavity and to
disease. complete the intended procedure.
• Hernia at site of entry (less than 1 in 100; uncommon).
• Thromboembolic complications (rare or very rare).
• Death; 3–8 in 100 000 women
8/30/17 ELBOHOTY
(very rare) undergoing
228
laparoscopy may die as a result of complications.

57
8/30/17

Bladder injury
Frequent risks • Prophylaxis
• The Royal College of Obstetricians and Gynaecologists advises
• are usually mild and self-limiting. that suprapubic insertion of the Veress needle should be
• Bruising avoided as it puts the dome of the bladder at risk of injury, and
carries a high failure rate.
• shoulder-tip pain • Similarly, insertion of secondary trocars should be performed
• wound gaping under direct view. Although not evidence-based, bladder
catheterisation prior to peritoneal insufflation and insertion of
• infection. trocars is recommended to avoid injury to a bladder distended
by urine.
• Insertion of an indwelling catheter in long procedures. Keeping
the bladder empty during surgery will protect it not only
because its decreased size will keep it out of the surgeon’s
8/30/17 ELBOHOTY 229
operating field,
8/30/17
but also because an empty bladder cannot230
ELBOHOTY
be
penetrated as easily as a distended one.

Intraoperative recogenition
• an obvious cystotomy or visualisation of urine leakage.
• A suspicion of a not so obvious injury may be raised by noting
haematuria or a distended catheter bag because of gas leaking
• Most injuries occur during dissection of the bladder through the defect into the bag. Therefore, it is always worth
from the cervix and therefore the most common site is inspecting the catheter and its bag, near the end of a complex
in the midline, above the inter-ureteric bar. laparoscopic pelvic operation and before closure.
• Intraoperative cystoscopy and/or instillation of 200– 300 mls of
• Less often the bladder can be put at risk during coloured saline (such as methylene blue or indigo carmine) into the
insertion of the Veress needle or a trocar. bladder will identify the site and extent of the injury.
• laparoscopic-assisted vaginal hysterectomy (LAVH), • Care is advised when instillating coloured saline to look for an injury,
as this may not be seen leaking intra-abdominally in cases where the
appear to be associated with a higher frequency of
bladder injury opens to the retro-pubic space (space of Retzius)
bladder injury compared with others. • Such an injury may occur for example during a difficult suprapubic
trocar insertion (previous suprapubic incision) which is accomplished
by repeated attempts. In such a case, an initial unsuccessful attempt
to insert the trocar may injure the bladder dome in a retro-peritoneal
8/30/17 ELBOHOTY 231 8/30/17 ELBOHOTY 232
fashion.

58
8/30/17

Postoperative
• suprapubic pain, haematuria, leakage of urine per vagina and oliguria.
• Most bladder injuries can be sutured in one or two layers using a 2-0 or 3-0
• Uroperitoneum can present with diffuse abdominal pain, distension and absorbable suture (such as polyglactin).
ileus. Characteristically, tenderness may be absent.The above symptoms
• A running non-locked repair with the sutures placed 0.5 to 1 cm apart and
and signs usually appear within the first 48 postoperative hours unless a
0.5 to 1 cm lateral to the cystotomy angles is suggested.
thermal injury has occurred.
• Alternatively, if extra-corporeal knotting is preferred, interrupted sutures can
• Thermal injuries may present after 10–14 days with uroperitoneum or
be used at 0.5 cm intervals, whereas a ‘figure of 8’ suture may be enough to
vesico-genital fistula.
close a small defect.
• Biochemistry investigations aid the diagnosis as serum creatinine levels will
• Injuries involving the trigone require additional attention. Repair should aim
be abnormally elevated due to reabsorption of urine creatinine through the
to avoid obstructing the ureters or the urethra. In such cases ureteral stents
peritoneal membrane.
must be inserted and the patency of the urethra and ureters confirmed
• A computed tomography (CT) scan with contrast may confirm the presence following repair.
of uroperitoneum and/or show direct evidence of an injury.
• A thermal injury to the bladder will require debridement before repair,
• Retrograde cystography will confirm the diagnosis and cystoscopy will whereas an injury that pierces the bladder through the space of Retzius
assess the injury and help decide whether conservative management is alone may be managed conservatively by an indwelling catheter for 2
appropriate, depending on the extent of the damage weeks.
8/30/17 ELBOHOTY 233 8/30/17 ELBOHOTY 234

precautions conservative management


• Ideally, bladder repairs should be watertight and leakage from
the suture line should be tested (for example with methylene • For a small bladder injury that is diagnosed
blue or indigo carmine). postoperatively
• A bladder catheter must be inserted and continuous
postoperative bladder drainage should be allowed for 2 weeks.
• Cystoscopic examination can assist in the
The above two measures (watertight closure and indwelling decision.
catheter) will improve healing and reduce the risk of • Antibiotics should be administered for 5–7
subsequent vesico-vaginal fistula formation.
• Prior to catheter removal, complete repair without leakage
days and an indwelling catheter kept for 2
should be confirmed by retrograde cystography. weeks.
• If contrast escape is noted then the catheter should be left in
situ and the test repeated in 1 week. Despite these measures, a
fistula can still form with an ELBOHOTY
8/30/17
approximate incidence of 5% (of
235
the 8/30/17 ELBOHOTY 236
cases where an injury occurred).

59
8/30/17

Ureteric injuries Sites


• The most common sites of ureteric injury in laparoscopic surgery are at the
• Rates as low as 0.06% (laparoscopic pelvic brim (where the ureter comes into close proximity with the infundibulo-
subtotal hysterectomies), and as high as pelvic ligament which contains the ovarian vessels) and lateral to the cervix
(during division or coagulation of the uterine artery or the uterosacral and
21% (deep infiltrating endometriosis cardinal uterine ligaments). Less often, injuries may occur at the ovarian
associated with hydronephrosis) fossa, for example during resection of endometriosis or ovarian remnants.

• Electrocautery inv in one quater of injuries

8/30/17 ELBOHOTY 237 8/30/17 ELBOHOTY 238

• laparoscopic procedures where a ureteric injury occurred in a • Ureteric stenting (including lighted stents) is useful
patient with severe endometriosis concluded that unconscious only in very select cases, where the pelvic anatomy is
acceleration of surgery, possibly caused by fatigue, contributed severely distorted and/or usual methods of ureter
to a judgement error that led to the injury. identification have failed
• the higher the body mass index the closer the ureter was found • adequate reflection of the bladder off the uterus and
to be to the cervix
the cervix during total laparoscopic hysterectomy will
move not only the bladder, but also the ureters away
from the uterine vessels and the cervix, thus reducing
the risk of injury.

8/30/17 ELBOHOTY 239 8/30/17 ELBOHOTY 240

60
8/30/17

Types of uretric injuries • Only a third of such injuries are recognised intraoperatively therefore any
uncertainty about the integrity of the ureter should prompt intraoperative
investigation and involvement of a urologist.
• Angulation • Cystoscopy allows visualisation of the ureteric orifices and urine jets which
rules out obstruction, but does not exclude other types of injuries.
• Crush • Presence of blood or air suggests injury. Intravenous administration of indigo
carmine colours the urine blue within 5 to 10 minutes and will assist a
• Ligation cystoscopic assessment as well as potentially allow the surgeon to identify a
urine leak laparoscopically.
• Thermal • Stents inserted without resistance, under direct laparoscopic visualisation to
ensure they do not exit through a possible injury, can also rule out
• Laceration obstruction. Occasionally, insertion of a stent alone can be therapeutic if the
problem was angulation (kinking) of the ureter. Ureteroscopy may locate the
• Transection The most commonly reported approximate height and extent of injury.
• Resection

8/30/17 ELBOHOTY 241 8/30/17 ELBOHOTY 242

• Flank pain and flank tenderness, • thermal injury to the ureter may result to delayed necrosis
and/or fistula formation that will often present clinically between
haematuria, oliguria or watery vaginal loss 10 and 14 days postoperatively.
may be present within the first 48 hours of • Ultrasound and/or CT scans can evaluate hydronephrosis,
an acute injury. urinomas and abscesses, whereas a CT intravenous urogram
(CT IVU) will locate the injury.
• Uroperitoneum will present clinically with • The consequences of an unrecognised injury can vary from
the often misleading features discussed spontaneous healing to fistula and/or stricture formation with
above.. associated deterioration of the function of the affected kidney.
This may occasionally require nephrectomy.
• A urinoma may develop • Up to 25% of unrecognised ureteral injuries result in eventual
loss of the ipsilateral kidney.
8/30/17 ELBOHOTY 243 8/30/17 ELBOHOTY 244

61
8/30/17

PRINCIPLES OF REPAIR
• 2-0 or 3-0 absorbable suture-polyglactin(vicryl)
• Thermal injury requires debridement
• Adequate but careful debridement to avoid shortening the ureter
(debridement may be needed to enable the use of the healthy ureter for re-
• Upper third-uretero ureterostomy,
anastomosis) • middle third-uretero urertosotomy or trans
• Adequate but careful dissection to avoid devascularisation
(dissection/mobilisation may be needed to lengthen the ureter for uretero uretrostomy
anastomosis)
• Anastomosis must be:
• Lower third-ureteroneocystomy with psoas


water-tight
tension-free
or boaris flap

• spatulated or fish-mouth

Use absorbable and intermittent sutures


• Ureteric repair-use absorbable n
• Avoid using too many sutures intermittent sutures...consider omental
• Use drainage (ureteral stents, bladder catheter, retro-peritoneal anastomotic
site drain)
flaps
• Consider omental flap to cover the repair site and increase vascularity
8/30/17 ELBOHOTY 245 8/30/17 ELBOHOTY 246
• When possible, repair by laparoscopy

Anaesthetic problems

8/30/17 ELBOHOTY 247 8/30/17 ELBOHOTY 248

62
8/30/17

Hysteroscopy
• Clinical negligence claims related to hysteroscopic procedures include
– complications related to uterine perforation
Injuries
– subsequent internal organ injury.
• The failure to recognise the complication is the commonest cause of
litigation.
• When there has been internal organ damage, such as bowel, patients may
remain asymptomatic for 2–10 days before the nature of the injury, often
thermal, becomes apparent.
• Factors that increase the risk of perforation include cervical stenosis, acute
anteversion or retroversion, lower-segment fibroids or intrauterine synechiae
and operator inexperience.
• Uterine injury without the use of an electrical source can usually be
managed by observation of signs of vaginal or intraperitoneal bleeding.
However, where an electrical source has been used, laparoscopy is advised
to rule out bowel injury.
• Fatal complications to which the clinician should be alert during
hysteroscopy include: fluid overload causing hyponatraemia and
subsequent respiratory arrest and seizures, air embolism leading to
collapse and death..

8/30/17 ELBOHOTY 249 8/30/17 ELBOHOTY 250

Bladder injury
• Most injuries occur from dissection of bladder
from cervix....mc site is midline..above
interuretric ridge
• Half bladder injuries remain unrecog intraop
• Uroperitoneum presents with abd distn, pain,
ileus but tenderness is ABSENT...usually
presents within 48 hrs after injury
• Thermal injuries present 10-14 days later
• Serum 8/30/17
creatinine raised
ELBOHOTY
as absorbed thru 251 8/30/17 ELBOHOTY 252

peritoneum

63
8/30/17

hysteroscopy Prerequisites

8/30/17 ELBOHOTY 253 8/30/17 ELBOHOTY 254

• An appropriately sized and fully


equipped treatment room (a
dedicated hysteroscopy suite) with
the necessary equipments.
• Appropriately trained and
motivated staff
• Proper patient selection
• Consent for the procedure.
• Guidelines and pathways
• Resuscitation equipment
8/30/17 ELBOHOTY 255 8/30/17 ELBOHOTY 256

64
8/30/17

Vaginoscopy versus
conventional hysteroscopy
• Vaginoscopy reduces pain during
diagnostic rigid outpatient
hysteroscopy.
Types • Vaginoscopy should be the standard
technique for outpatient hysteroscopy

8/30/17 ELBOHOTY 257 8/30/17 ELBOHOTY 258

Preparation
8/30/17 ELBOHOTY 259 8/30/17 ELBOHOTY 260

65
8/30/17

• Explain procedure and


reassure patient
• Place patient in dorsal
lithotomy position Analgesia
• Wash with warm
disinfectant and drape
8/30/17 ELBOHOTY 261 8/30/17 ELBOHOTY 262

• Avoid use of opiate analgesia


• Instillation of local anaesthetic into
before outpatient hysteroscopy.
the cervical canal does not reduce
pain during diagnostic outpatient
• Use NSAIDs around 1 hour before hysteroscopy but may reduce the
their scheduled outpatient incidence of vasovagal reactions.
hysteroscopy appointment. • Topical application of local
anaesthetic to the ectocervix
should be considered where
application of a cervical tenaculum
8/30/17 ELBOHOTY 263 is necessary.
8/30/17 ELBOHOTY 264

66
8/30/17

Cervical preparation Choice of hysteroscopy

No evidence of benefit in terms of reduction of pain,


rates of failure or uterine trauma.
8/30/17 ELBOHOTY 265 8/30/17 ELBOHOTY 266

size and angle of


hysteroscope Angle of objective Lens
• Miniature hysteroscopes (2.7mm with a 3–
3.5mm sheath) should be used for
diagnostic outpatient hysteroscopy
• 0°
• Choice of hysteroscope should be left to the
discretion of the operator. • 12°
Increased
• 15of°
field
view for
• 25 °
12-30 °
• 30 °

8/30/17 ELBOHOTY 267 8/30/17 ELBOHOTY 268

67
8/30/17

Negotiating the cervical canal


Hysteroscopy Flexible Rigid
types hysteroscopes hysteroscope
Advantages less pain 1. better images
2. fewer failed
procedures
3. quicker
examination
time
4. reduced cost

8/30/17 ELBOHOTY 269 8/30/17 ELBOHOTY 270

8/30/17 ELBOHOTY 271 8/30/17 ELBOHOTY 272

68
8/30/17

Methods of fluid instillation


Distention media of the uterine
cavity
• Pressure cuff
• CO2 distention – Altering the pressure in the cuff can vary fluid
infusion rate.
• Saline
• Glycine • An electric pump
– It is usually set up at 70 mmHg.

8/30/17 ELBOHOTY 273 8/30/17 ELBOHOTY 274

Medium Media
Advantagesused in hysteroscopy
Disadvantages
CO2 • Well-tolerated, convenient and
easy to use
• Most serious disadvantage is
that of gas embolism
Hysteroscopic sterilisation
• Visibility can be distorted in the
presence of bleeding The procedure involves placing a micro-insert, consisting of a
Shoulder tip pain

stainless steel inner coil with polyester fibres and a super-
Normal
saline


Low cost and readily available
Reduce vasovagal episodes
Improved image quality and allows
outpatient diagnostic hysteroscopy
elastic nitinol outer anchoring coil, into the intramyometrial
• Also used for operative to be completed more quickly portion of each fallopian tube
hysteroscopy in bipolar compared especially in the
electrosurgical operative presence of bleeding
hysteroscopy (e.g. Versapoint)
1.5% • Electrolyte-free and non- • Excessive absorption causes
glycine conductive hyponatremia.
• Used in monopolar • Severe overload can lead to
electrosurgical operative haemolysis, coma and death
hysteroscopy

no significant difference between the pain experienced with the


use of carbon8/30/17
dioxide versus ELBOHOTY
normal saline for outpatient 275 8/30/17 ELBOHOTY 276
hysteroscopy

69
8/30/17

Feasability
• Successful bilateral placement of the Essure"
device is possible in 81–99% of cases.
• The average time taken toinsert the device in
both fallopian tubes varied from 10.0 to 13.8
minutes.
• Sterilisation was shown to be 99.83% effective
based on a 5-year clinical study.
• The newly released long- term follow-up of a
phase III study observed no pregnancy
8/30/17 ELBOHOTY 277 8/30/17 ELBOHOTY 278
following hysteroscopic sterilisation

Causes of faliure
• Non-compliance
• Misinterpretation of the hysterosalpingogram
• Luteal phase pregnancy
• Failure to use contraception

8/30/17 ELBOHOTY 279 8/30/17 ELBOHOTY 280

70
8/30/17

• Postoperative pain – 79%


Complications
• Vasovagal syncope: High incidence is that gynaecologists performing the procedure have had a Other complications
steeper learning curve; with the use of smaller diameter scopes, the incidence of vaso-vagal syncope is
reduced.
• Cervical/vaginal/prolonged bleeding • incidence of relook hysteroscopy for this is
• Thrush/infection
• Expulsion rate: 0.04–3.0%.It is more likely to happen in a proximally placed
0.04% (20/545 cases). Only 7 out of the
device which then migrates into the cavity. Most of the expulsions happen 20 cases found structural abnormality.36
before the confirmation of bilateral occlusion.
• Perforation:1% and 2%.
There has been no report of serious
• Abdominal migration: 0.04%. These women can be asymptomatic but may also present with adverse effects of Essure" such as death,
symptoms of bowel obstruction. The diagnosis can be made at the 3-month HSG
• Nickel allergy: 14–18% of women. There have been cases of allergic reactions such as bowel injury or vascular injury.
urticaria and erythema, which resolved following removal of the device.The device was removed if the
conservative measures were not successful.

• Chronic pelvic pain: 2.1–24% Device removal has to be considered where the pain does not
resolve with conservative measures of pain management. Women should be reassured initially and managed
conservatively as the pain will resolve in 50% of cases in 3 months.40 Where the device cannot be visualised
or removed by hysteroscopy, such cases need to be dealt with by laparoscope with or without intraoperative
fluoroscopy to identify the device. Salpingectomy was the most commonly performed technique for device
8/30/17 ELBOHOTY 281 8/30/17 ELBOHOTY 282
removal.
• 5–12% of women were noted to have heavy bleeding during the 5-year follow-up period.

Radiological follow-up

• In the UK and Europe the placement of an


Essure" device can be followed up by
performing a pelvic ultrasound scan at 3
months post-procedure

• Hysterosalpingography performed at 3 months showing appropriate


placement of Essure" device: (a) proximal end of outer coil; (b) proximal
8/30/17 ELBOHOTY 283 marker of inner coil; (c) distal marker
8/30/17 of outer coil; (d) distal end of inner
ELBOHOTY 284
coil. Reproduced with permission from Bayer.

71
8/30/17

• Transvaginal ultrasound of pelvis in transverse Role in infertility


view showing bilateral Essure" device placed • Laparoscopic salpingectomy or tubal occlusion by
Filshie clip in the management of hydrosalpinx has
correctly in both fallopian tubes.
been shown to improve the pregnancy rate after in
vitro fertilisation treatment.
• Essure" can be an alternative option in cases with
adhesions, pelvic inflammatory disease, Crohn’s
disease, endometriosis or laparotomies where the risk
of laparoscopic salpingectomy or occlusion carries a
significant risk.
• rate of successful tubal occlusion of 98.1%. Live
pregnancy rates per embryo transfer are 28%.
• No adverse effect on fetuses has been reported.
8/30/17 ELBOHOTY 285 8/30/17 ELBOHOTY 286

MRI Approach to stenotic cervix


• The Essure" device is magnetic
resonance-conditional according to the
terminology specified by the American
Society for Testing and Materials
standards.
• This means a woman with an Essure"
device can undergo MRI scanning with
static magnetic field of 3 tesla or less and
maximum spatial gradient of magnetic field
of 7208/30/17
gauss/cm or less.
ELBOHOTY 287 8/30/17 ELBOHOTY 288

72
8/30/17

Complications

8/30/17 ELBOHOTY 289 8/30/17 ELBOHOTY 290

•Entry Distension media


- Faliure
- False passage
- perforation Fluid overload: prevented by keeping the operating
time
to minimum and use of isotonic saline
- Avoid entering vascular channels.
- Close monitoring of fluid balance.

8/30/17 ELBOHOTY 291 8/30/17 ELBOHOTY 292

73
8/30/17

What is the definition of fluid


overload?
• A fluid deficit of more than 1000 ml should • intrauterine pressures > 75 mm Hg
be used as threshold to define fluid increases the volume of media passing
overload when using hypotonic solutions back along the fallopian tubes and into the
in healthy women of reproductive age. peritoneal cavity
• A fluid deficit of 2500 ml should be used
as threshold to define fluid overload when
using isotonic solutions in healthy women
of reproductive age.

8/30/17 ELBOHOTY 293 8/30/17 ELBOHOTY 294

presentation management
• Where excessive systemic absorption of fluid distension media
is suspected, strict fluid balance monitoring should be
• Surgeons should be cognisant of commenced, a urinary catheter inserted and serum electrolytes
cardiovascular and neurological symptoms measured. If the patient develops signs of cardiac failure or
associated with systemic absorption of pulmonary oedema a cardiac echocardiogram and chest X-ray
should be undertaken. [GPP]
fluid distension media complications to
• Asymptomatic hypervolemia with or without hyponatraemia
allow timely recognition and treatment. should be managed by fluid restriction with or without diuretics.
[GPP]
The management of symptomatic hypervolemic hyponatraemia
• requires multidisciplinary involvement including anaesthetists,
physicians and intensivists in a high dependency or intensive
care unit. Initial treatment with 3 % hypertonic sodium chloride
8/30/17 ELBOHOTY 295 infusion 8/30/17 ELBOHOTY 296

74
8/30/17

Gas embolism

8/30/17 ELBOHOTY 297 8/30/17 ELBOHOTY 298

• The pathophysiology involved in an embolic phe- nomenon is the passage of


• Clinically significant gas or air embolism is a the embolus to the pulmonary circulation, initially creating a high ventilation/
rare complication of hysteroscopy. However perfusion ratio which reduces the end tidal carbon dioxide.
• This causes diversion and overperfusion of the pulmonary blood flow to the
this diagnosis should be considered if the unaffected areas within the lung, away from the embolised area leading to a
patient develops sudden oxygen desaturation low ventilation/perfusion ratio causing hypoxaemia.
• In addition the physical presence of the embolus can cause mechanical
or cardiovascular collapse during the obstruction which could lead to cardiac strain and cardio- vascular collapse.
procedure In this situation the patient needs to be resuscitated and managed in an
intensive care setting.

8/30/17 ELBOHOTY 299 8/30/17 ELBOHOTY 300

75
8/30/17

Procedure itself Hysteroscopic Uterine


perforation management
- Infection
- Bleeding
- Cervical/uterine damage/perforation

8/30/17 ELBOHOTY 301 8/30/17 ELBOHOTY 302

8/30/17 ELBOHOTY 303 8/30/17 ELBOHOTY 304

76
8/30/17

Cystoscopy

8/30/17 ELBOHOTY 305 8/30/17 ELBOHOTY 306

Indications
• visible and unexplained haematuria, either without urinary tract infection
(UTI) or that persists after successful treatment of UTI in people aged 45
years and over
• visible and unexplained haematuria with a raised white cell count on blood
test in people aged 60 years and over • The only true contraindication to
• dysuria with unexplained, non-visible haematuria in people aged 60 years cystoscopy is an untreated UTI, outlined in
and over
• recurrent UTI
the British Association of Urological
• bladder pain syndrome Surgeons (BAUS) guidelines
• voiding symptoms
• vesicovaginal or colovesical fistulae
• urethral stricture
• congenital genital tract anomalies.
• Cystoscopy is part of some operations, such as: mid-urethral slings and
colposuspension intravesical botox injections
staging of gynaecological cancers.
8/30/17 ELBOHOTY 307 8/30/17 ELBOHOTY 308

77
8/30/17

Risks
• Common risks (greater than 1 in 10)
– mild burning or bleeding on passing urine for a short period after the
operation
– biopsy of abnormal areas in bladder
• Occasional risks (between 1 in 10 and 1 in 50)
– infection of the bladder requiring antibiotics Rare (less than 1
in 50)
– temporary insertion of a catheter
– delayed bleeding requiring removal of clots or further
surgery
• – injury to the urethra causing delayed scar formation
• very rarely,
– perforation of the bladder requiring a temporary catheter or open surgical
repair
8/30/17 ELBOHOTY 309 8/30/17 ELBOHOTY 310

8/30/17 ELBOHOTY 311 8/30/17 ELBOHOTY 312

78
8/30/17

Genital trauma

8/30/17 ELBOHOTY 313 8/30/17 ELBOHOTY 314

Uterine perforation
Technique and suture
Site of injury Technique Suture Needle

anal canal continuous or 3-0 polyglactin Rounded body


mucosa interrupted
technique
IAS muscles interrupted or 3-0 PDS or 2-0
mattress polyglactin
sutures
full thickness either an
external anal overlapping or
sphincter (EAS) an end-to-end
tear
partial an end-to-end
thickness (all 3a technique
and some 3b)
tears

8/30/17 ELBOHOTY 315 8/30/17 ELBOHOTY 316

79
8/30/17

Incidence

• Factors increasing risk of uterine perforation:


– Uterine anomalies • Average incidence 0.002 – 1.7%
– Infection
– Recent pregnancy
– Menopause
• TOP is the most common procedure associated
with uterine perforation.
8/30/17 ELBOHOTY 317 8/30/17 ELBOHOTY 318

Potential Sequelae Potential Sequelae


• There is a danger with any perforation that an • History of previous perforation should be
sought early in pregnancy, as many severe
instrument may have passed through the abdominal pain antenatally may represent a
perforation and damaged an abdominal organ, uterine dehiscence or rupture.
such as the intestine, ureter, urinary bladder or • Cases of uterine rupture during labour has
a major blood vessel. been reported.
• The majority of cases that resulted in rupture
• Up to 15% of uterine perforations caused by fitting had their initial perforation caused by
an IUCD involve a pelvic or abdominal viscera, operative hysteroscopic procedure for
division of adhesions & had used monopolar
• With 3 – 7.5% involve bowel injury. diathermy.
8/30/17 ELBOHOTY 319 8/30/17 ELBOHOTY 320

80
8/30/17

Recognition Common Sites


• Early recognition & proper management
significantly reduces morbidity, long-term Anterior wall 40%
sequelae & mortality.
• Initially, an injury can be suspected if Cervical canal 36%
extension of the instrument goes beyond the
limitation of the uterus. Right lateral wall 21%
• Furthermore, loss of resistance with further Left lateral wall 17%
instrumentation is highly indicative.
• Sudden loss of vision in hysteroscopy due to Posterior wall 13%
collapse of uterus & bleeding together with loss
of distending medium is highly suggestive. Fundus 13%
• Finally, direct visualisation of the perforation TOTAL 140%
site, omentum
8/30/17 or bowel is diagnostic.
ELBOHOTY 321 8/30/17 ELBOHOTY 322

Incidence with various


instruments Risk Factors
Suction cannula 51%
• Uterine & cervical risk factors
– Concerning the uterus & cervix condition
Hegar dilator 25%
• Surgical risk factors
– Concerning the surgical procedure itself
Curette 16%
• Surgeon’s risk factors
– Concerning the operating surgeon.
Others 8%
8/30/17 ELBOHOTY 323 8/30/17 ELBOHOTY 324

81
8/30/17

Uterine & Cervical Risk Factors Surgical Risk Factors


• Advanced gestation when TOP is done
• Parous uterus • ERPC for PPH
• Recent pregnancy in past 6 months • Biopsy collection in presence of intrauterine
• Postmenopausal small uterus or tight cervix adhesions in postmenopausal bleeding
• Uterine cavity distended by fibroids investigation
• Intrauterine synechiae or adhesions
• Pyometra or infection
• Position of uterus: Surgeon’s Risk Factors
– Retroverted, • Junior trainees have 5-fold increase than
– Acutely anteverted or senior staff
– Retroflexed uterus
– A trained surgeon is also more likely to
• Uterine anomalies recognise a perforation early before other
• Scarred uterus
8/30/17 ELBOHOTY 325
damage occurs
8/30/17 ELBOHOTY 326

Prevention Prevention
• Risk assessment
• Adequate preparation • Bi-manual examination correctly identifying
– Prostaglandins or misoprostol in premenopausal uterine size, position & attitude
women
– No benefit in postmenopausal
• Gradual cervical dilatation by ½ size dilators
• Option of medical TOP would reduce risk of • Experienced operator
perforation particularly in the 2nd trimester • Ultrasound guidance in experienced hands
• Accurate estimation of gestational age
• Correct equipment • Laparoscopic guidance (if an abdominal
– Use of tapered Hawkins-Ambler dilators require procedure is done at same time)
less force then the parallel-sided Hegar dilators that
are in common use in NHS units in UK
8/30/17 ELBOHOTY 327 8/30/17 ELBOHOTY 328

82
8/30/17

• What is the more frequent location of major


vessel injury during umbilical trocar
insertion?
a. Bifurcation of aorta
b. Right common iliac artery
c. Right common iliac vein
d. LEFT COMMON ILIAC ARTERY
e. Left common iliac vein d

8/30/17 ELBOHOTY 329 8/30/17 ELBOHOTY 330

12 . Unrecognised ureteric injury during • The answer is E 60%.


laproscopy: • 2/3 of cases
A .10%

B.15%

C .30%
D.40%

E. 60%


8/30/17 ELBOHOTY 331 8/30/17 ELBOHOTY 332

83
8/30/17

13 . During laparoscopy for deep • The answer is A


infiltering endometriosis ureteric injury • Most published studies quote a range of
is: ureteric injury rates at laparoscopic
A.1in 5
 gynaecological surgery from <1% to 2%.,
B .1in10 and as high as 21% (deep infiltrating
C.1in 20
 endometriosis associated with
hydronephrosis) have been reported.
D. 1in 25

E.1in 30

8/30/17 ELBOHOTY 333 8/30/17 ELBOHOTY 334

31. A postop MEOWS chart was given: 65 yrs, known type 2 diabetic on metformin,
PH of DVT. Undergone TAH + BSO for endometrial cancer at 12:00. You were
called at 17:00 as the nurse was concerned about her chart & the patient is
complaining of lower abdominal pain & breathlessness.
At 17:00 exact the vitals were: BP 110/70 [It was 90/60 at 16 ], T 37°C, pulse rate • e
110 beat/ minute, RR 20/ minute, O2 sat 100%, pain score 5, fully alert. Her
postop Hb is 104%.
What is the most likely underlying problem?
A. PE.
B. Pelvic sepsis.
C. DKA.
D. Opiate overdose.
E. Internal bleeding.

8/30/17 ELBOHOTY 335 8/30/17 ELBOHOTY 336

84
8/30/17

– 32. What is the commonest type of ureteric injury • d


after therapeutic laparoscopy?
– A. Crushing.
– B. Resection.
– C. Angulation.
– D. Transection
– E. Coagulation

8/30/17 ELBOHOTY 337 8/30/17 ELBOHOTY 338

33. What is the commonest vascular injury during "umbilical" primary trocar insertion?
A. Suprarenal aorta.
B. Inferior vena cava.
C. Inferior epigastric.
D. Left Common iliac artery
E. Internal ileac. • d

8/30/17 ELBOHOTY 339 8/30/17 ELBOHOTY 340

85
8/30/17

• 34. A scenario of TAH for a large fibroid which was found in the broad ligament but they
asked: from the following, what best describes the course of the ureter?
–A. The ovarian vessels course posterior to the ureter.
• e
–B. The ureter courses above the uterine artery.
–C. 1/3PrdP of the ureter is in the abdomen.
–D. The ureter courses medial to the bifurcation of the common ileac.
–E. The ureter courses medial to the internal ileac.

8/30/17 ELBOHOTY 341 8/30/17 ELBOHOTY 342

36. During hysteroscopic polypectomy, perforation was identified after avulsion of


the polyp. There was no active bleeding from perforation site & the woman is
stable.
What is the most appropriate management?
A. Laparotomy. • d
B. Laparoscopy.
C. Observation.
D. Antibiotics + observation.
E. Reassure & discharge as usually planned after diagnostic procedures.

8/30/17 ELBOHOTY 343 8/30/17 ELBOHOTY 344

86
8/30/17

• A 55-year-old woman is due to come in for total


abdominal hysterectomy and bilateral salpingo-
oophorectomy for a large mucinous ovarian
cyst. She takes sequential HRT for menopausal
symptoms. What is the approximate overall risk • The correct answer is 4 operations in
of serious complications from abdominal every 100. The overall risk of serious
hysterectomy? complications from abdominal
• 1 operation in every 100 hysterectomy is approximately four women
• 2 operations in every 100 in every 100 (common).
• 3 operations in every 100
• 4 operations in every 100
• 5 operations in every 100
8/30/17 ELBOHOTY 345 8/30/17 ELBOHOTY 346

• A woman has been recommended to undergo


hysterectomy and bilateral salpingo-oophorectomy
for benign disease. You discuss the risks and
benefits of an open versus a laparoscopic
procedure. • The correct asnwer is urinary tract injury.
• Which sort of injury is more common at Laparoscopic surgery involves risks to bowel,
laparoscopic hysterectomy compared to an open urinary tract and major blood vessels. These
procedure? risks are higher in women who are obese or
significantly underweight, however the risks
• Bowel of laparotomy are significantly greater in the
• Nerve morbidly obese. Urinary tract injury and
• Ovary vaginal cuff dehiscence are more common in
• Urinary tract the laparoscopic approach with an odds ratio
of 2.61 for urinary tract injury
• Vascular
8/30/17 ELBOHOTY 347 8/30/17 ELBOHOTY 348

87
8/30/17

• A 36-year-old woman attends the antenatal


clinic at 20 weeks of gestation. She has had
three previous caesarean sections and has a
normal placental site. She consented for • The correct answer is blood transfusion.
another caesarean section. What is the most Elective repeat caesarean section is
likely surgical complication? associated with increasing risks that rise
• Bladder injury with each successive pregnancy. Blood
• Blood transfusion transfusion rises from 7.9% with a third
• Bowel injury caesarean section to 14.1% with the fifth
caesarean.
• Fetal laceration
• Hysterectomy
8/30/17 ELBOHOTY 349 8/30/17 ELBOHOTY 350

A patient is seen on the second post operative day after a difficult abdominal hysterectomy
complicated by haemorrhage from the left uterine artery pedicle. Multiple sutures were
placed into this area to control bleeding. The patient now has fever, left back pain, left Single Best Answer
costovertebral angle tenderness and haematuria. An ultrasound examination shows that
fluid has accumulated in the left flank. A ureteral injury is diagnosed. Options:
If the injury had been recognized at the time of surgery, which of the following procedures
could have been recommended?

Options:
a) Percutaneous nephrosto9my
b) Placement of ureteral stent without anastomosis
a) Percutaneous nephrosto9my
b) Placement of ureteral stent without anastomosis c) Intraperitoneal drainage without anastomosis
c) Intraperitoneal drainage without anastomosis
d) Uretero-ureteral anastomosis
d) Uretero-ureteral anastomosis
e) Ureteral reimplantation into the bladder e) Ureteral reimplantation into the bladder

8/30/17 ELBOHOTY 351 8/30/17 ELBOHOTY 352

88
8/30/17

A. Abandon procedure & plan future The surgeon has opened the young girl as
management acute appendicitis. The signed consent
B. Abandon procedure included laparoscopy + removal of the
C. B. Proceed as planned/consented for appendix ± laparotomy. The appendix was
C. Ask a collegue found normal but there was an uncomplicated
D. Meet with next of kin 3 cm cyst [y] on the Rt. ovary [x]. The surgeon
E. Remove X & Y decided to leave the appendix & called you for
F. Remove Y from X the cyst.
G. Take biopsy from Y & close
H. Remove
8/30/17 Y. ELBOHOTY 353 8/30/17 ELBOHOTY 354

I. Other options

• A woman is undergoing laparoscopic • What is the most common site for


hysterectomy for HMB, signed for laparoscopy + uterine perforation during evacuation?
removal of the uterus ± laparotomy. A partially • A. Ant. wall
soild and cystic fungating ovarian mass [y] was • B. Post. wall
found on the Right ovary [X].
• C. Cx canal
• D. Fundus
• E. None.

8/30/17 ELBOHOTY 355 8/30/17 ELBOHOTY 356

89
8/30/17

*Vascular injury
• Inferior epigastric artery is a branch of: A. Superior gluteal
• A. Ext. iliac
B. inferior gluteal
• B. Int. iliac
• C. Femoral
C. ovarian
• D. Int. thoracic D. uterine
• E. Popliteal E.internal pudendal
F. internal iliac
8/30/17 ELBOHOTY 357 8/30/17 ELBOHOTY 358

140 . Patient has undergone laproscopic • C ovarian


salpingectomy for ectopic pregnancy
in the recovery room patient devolved
hypovolemic shock and prepared for
laprotomy .

8/30/17 ELBOHOTY 359 8/30/17 ELBOHOTY 360

90
8/30/17

141 . Patient during VD had 4th degree tear • E.internal pudendal


and massive bleeding

8/30/17 ELBOHOTY 361 8/30/17 ELBOHOTY 362

142 .Patient bleed after sacrospinous • E.inferior gluteal


fixation

8/30/17 ELBOHOTY 363 8/30/17 ELBOHOTY 364

91
8/30/17

* unexpected pathology 143 . Patient consented for laproscopic


A. abundant and medical ttt hysterectomy with past history of dermoid
B.abundant and further assessment cyst removal.
Laparoscopic salpingectomy Intra op surgeon find dermoid cyst X 4 cm
D. remove x from y adherent to the pelvic wall in the ovary Y.
E.remove x &y
F. laprscopic biopsy and abundant
G. go as planned
8/30/17 ELBOHOTY 365 8/30/17 ELBOHOTY 366

144 . 31 years lady had a laproscopy for


suspected appendicitis however her
• D. remove x from y
appendix looked health with torted ischemic
Right fallopian tube with watery discharge.

8/30/17 ELBOHOTY 367 8/30/17 ELBOHOTY 368

92
8/30/17

• Laparoscopic salpingectomy If there is *Post hysterectomy complication


detortion in the option it will be better A. wound infection
B. chest infection
C. UTI
D.infected vault haematoma
E. vault haematoma
F. bowel injury
G. check fluid blance
h. active bleeding
8/30/17 ELBOHOTY 369 I . ureteric injury
8/30/17 ELBOHOTY 370

145 55 year old lady she is known to be • chest infection


heavy smoker. She is presented 48
hrs post hysterectomy with fever temp
39 ,tachycardia and tachypnea

8/30/17 ELBOHOTY 371 8/30/17 ELBOHOTY 372

93
8/30/17

146 . 3 days post op not recovered well • ureteric injury


ask for analgesia with abdominal and
back pain not febrile mild tachycardia
poor urine out put.

8/30/17 ELBOHOTY 373 8/30/17 ELBOHOTY 374

147 . 12 hrs post surgery pulse 100 BP • active bleeding


90/45 poor urine out put but said in
3hrs its 80 ml O/E tender abdomen..

8/30/17 ELBOHOTY 375 8/30/17 ELBOHOTY 376

94
8/30/17

148 . The commonest site for uterine • a


perforation during surgical evacuation
A.anterior wall
B.posterior
C. cervical
D.fundus

8/30/17 ELBOHOTY 377 8/30/17 ELBOHOTY 378

149 . What you do to reduce risk of • b


uterine perforation during evacuation
A. straiten the Cx caudally
B. done under US guidance
C. less Cx dilatation

8/30/17 ELBOHOTY 379 8/30/17 ELBOHOTY 380

95
8/30/17

• 131. 9 years girl came with her parent to


the ER with sudden onset of Lt iliac fossa
pain with nausea and vomiting and • The answer is: D. suspect torsion and
leucocytosis what is your next step in the prepare for laparoscopy
management?
• A. analgesia and observation

• B. suspect cyst torsion and give analgesia

• C. suspect appendicitis and send for surgery

• D. suspect torsion and prepare for
laparoscopy

8/30/17 ELBOHOTY 381 8/30/17 ELBOHOTY 382

• Patient with history of subfertility and PID • The answer is: A. acute appendicitis
present with Rt iliac fossa pain nausea and
vomiting .leucocytosis 19.000 CRP 20 US non
compressible mass 5cm in length and 10 mm
in diameter what is the diagnosis:
• A. Acute appendicitis
• B. Fallopian tube infection
• C. Pelvic abscess
• D. Ectopic pregnancy

8/30/17 ELBOHOTY 383 8/30/17 ELBOHOTY 384

96
8/30/17

EMQ Options
• Azithromycin
• Cefazolin .
• Amoxaciline + clavuronic acid
• Ceftriaxone i/m then doxy & metro
• Clinda + metro.
• Ofloxacin + metro
• Cefazoline + gentamicin
• only one option of flucloxacillin [& it was i/v, qds]
• Many other options including benzyl penicillin,…..
8/30/17 ELBOHOTY 385
• No need
8/30/17
for Antibiotic
ELBOHOTY 386

Questions: select for each scenario the most


EMQ. Suture material
appropriate antibiotic
Option Product Description Needle Gauge

1. A 55 year old woman is planned for TAH. She has A Coated vicryl ® Braided polyglactin Half circle, roundbodied 3-0

a history of PID & adhesions 22 years back. B

C
Coated vicryl ®

Coated vicryl ®
Braided polyglactin

Braided polyglactin
J needle, roundbodied, heavy

Half circle, roundbodied


2-0

2. A 44 year old woman is planned for VH and is D Dermabond ® Topical skin adhesive None

known to have mitral regurge E Endoloop® ligature Braided polyglactin None 0


F Mersilene ® Polyester fibre Blunt point, halfcircle, double 5mm
3. A 60 year old woman with postmenopausal G Monocryl ® Monofilament: polypropylene Half circle, roundbodied 1

bleeding is planned to have Hysteroscopy H Nylon Monofilament Straight cutting 0


1 PDS Monofilament Halfcircle, roundbodied 3-0
4. A postpartum woman diagnosed with painful J PDS Monofilament: looped Blunt, taperpoint, half circle, heavy 0

breast with fever and examination revealed that K Prolene® Monofilament: polypropylene Curved double 6-0

she has mastitis. L


M
Prolene®
Stainless steelwire
Monofilament: polypropylene
Multifilament
Curved, reverse cutting
Tapercut, roundbodied
4-0
2 mm

5. Young, non pregnant, diagnosed with acute PID. N Steristrips ® Skin adhesive strips None

She is8/30/17
allergic to penicillin.
ELBOHOTY 387
O VIcrylRapide®
8/30/17
Braided polyglactin
ELBOHOTY
Tapercut, halfcircle 2-0
388

97
8/30/17

1. After encountering a 3rd degree perineal tear, you identified both IAS & EAS,
you are about to suture them using overlap method.
2. You are about to close the sheath for a 72 years old woman after the oncology
consultant have finished the staging laparotomy for an advanced ovatrian
tumour . It was midline laparotomy and you intend to do mass closure of – EMQ. The most likely postop complication.
anterior Abdominal wall
–Options:
3. You are about to suture the episiotomy skin.
–A. Acute urine retention.
–B. Intraabdominal hemorrhage.
–C. Bladder injury.
–D. Ureteric injury.
–E. Bowel injury.
–F. ? Many others

8/30/17 ELBOHOTY 389 – 44. 8/30/17 ELBOHOTY 390

EMQ. The most appropriate/useful investigation


Options:
A. CXR.
B. XR of the abdomen/pelvis.
– Day 2 post diagnostic laparoscopy. C/O C. CT scan of the pelvis.
generalized abdominal pain & distention, nausea, D. CT scan of the pelvis, abdomen & chest.
E. MRI pelvis
vomiting & diarrhea. MRI abdomen & pelvis
F. US scan of the pelvis.
G. Hysteroscopy.
H. HSG.
– Day 13 post difficult laparoscopy for I. Endometrial biopsy.
endometriosis. C/O Rt loin pain & hematuria. J. Pelvic angiography.
K. ? Many others.

46. A woman was found to have a large fibroid. US showed a larg fibroid in the
upper part of the uterus. The cavity was distorted & not fully seen. The
woman opted for UAE.

47. During surgery for a large fibroid, a biopsy was taken from a degenerative part
8/30/17 ELBOHOTY 391 & showed8/30/17
deeply infiltrating leiomayosarcoma.
ELBOHOTY 392

98
8/30/17

– A woman admitted that she was exposed to genital


cutting as young. You examined her & found the
clitoris, labia minora & labia majora are removed.
– What type of WHO FGM classification is this?
–A. Type 1.
–B. Type 2.
–C. Type 3.
–D. Type 4.
–E. Type 5
8/30/17 ELBOHOTY 393 8/30/17 ELBOHOTY 394

• What is the commonest complication of


surgical evacuation?
• A. Peroration Operative Techniques
• B. Infection
• C. Bleeding requiring BT
• D. Cx injury/laceration
• E. Effects on future fertility.

8/30/17 ELBOHOTY 395 8/30/17 ELBOHOTY 396

99
8/30/17

• The answer is undertake supplemental


action. In robotic surgery, the surgeon
Robotic surgery is increasingly becoming important in
controls the instruments, camera and
gynaecological surgery. What is the main role of an assistant in
robotic surgery? energy source remotely from hand and
foot controls at the consoles. A bedside
assistant is utilised for supplemental
Adjustment of the 3D vision
actions, such as suction, retraction and
Control of the camera uterine manipulations.
Help manipulate the instruments • Nair R, Killicoat K, Ind TEJ. Robotic
Provide additional foot control at the console surgery in gynaecology. The Obstetrician
& Gynaecologist2016;18:223–31.
Undertake supplemental action
8/30/17 ELBOHOTY 397 8/30/17 ELBOHOTY 398

What is the main advantage of robotic surgery over laparoscopic • The answer is better precision and
surgery? microsurgical dissection. Compared with
conventional laparoscopy, the robotic
system downscales movements to up to
Better precision and microsurgical dissection 10 times, which provides tremor filtration
Decreased hospital stay and allows for precise movements. A
stable camera with 3D vision further
Improved cosmesis assists such precision and microsurgical
dissection.
Less pain after surgery
• Nair R, Killicoat K, Ind TEJ. Robotic
Quicker recovery for the patient surgery in gynaecology. The Obstetrician
& Gynaecologist2016;18:223–31.
8/30/17 ELBOHOTY 399 8/30/17 ELBOHOTY 400

100
8/30/17

• In primary CS, most bladder injuries occur


You are performing a caesarean section (CS) on a 29-year-old
primigravida at 38 weeks of gestation because the fetus is large for during peritoneal entry, while in repeat CS
gestational age and the woman has poorly controlled diabetes. When most occur during dissection of the
is the bladder most likely to be injured during this procedure? bladder from the lower uterine section (i.e.
raising the bladder flap).
During closure of the second layer • Field A, Haloob R. Complications of
During entry into the peritoneal entry caesarean section. The Obstetrician &
Gynaecologist 2016; DOI:
During its dissection from the lower uterine segment 10.1111/tog.12280.
When closing the visceral peritoneum
When securing the angles
8/30/17 ELBOHOTY 401 8/30/17 ELBOHOTY 402

• The classic presentation of Ogilvie syndrome


is progressive abdominal distension, which
A 30-year-old had an emergency caesarean section in the second
stage for a persistent bradycardia 6 days ago. She now presents with
may initially be painless and associated with
progressive abdominal distension, which was initially painless but varying degrees of constipation. As the
has become increasingly painful. The pain is localised mainly to the
right side. She is tachycardiac on examination and also pyrexial. caecum becomes more dilated, the pain
What is the most likely diagnosis?
worsens, localising to the right-hand side,
Bowel obstruction from adhesions and is associated with tachycardia.
Bowel perforation Eventually there is caecal ischaemia,
Faeculent peritonitis
perforation and peritonitis.
Infected haematoma
Ogilvie syndrome • Field A, Haloob R. Complications of

caesarean section. The Obstetrician &
Gynaecologist2016;DOI: 10.1111/tog.12280
8/30/17 ELBOHOTY 403 8/30/17 ELBOHOTY 404

101
8/30/17

A 30-year-old woman presents 7 days after an emergency caesarean • POVT presents with abdominal pain, pyrexia,
section with abdominal pain and a fever of 37.7°C . This isassociated nausea, vomiting, malaise and ileus with the fever
with nausea, vomiting and mild abdominal distension. She is started persisting despite antibiotics. On deep palpation
on antibiotics and sent home but 3 days later she re-presents with there is typically a mass in the adnexa that
no improvements in her symptoms and features of paralytic ileus.
represents the thrombosed vein surrounded by an
She is examined and found to have a tube-like mass in her abdomen
on deep palpation. A request is made for an ultrasound scan of the inflammatory mass – this is found in approximately
abdomen and pelvis. What is the most likely cause of this woman's 50% of cases. Most cases present within 10 days
symptoms? postnatally and the palpated mass is typically
tube-like. Differential diagnoses include
Infected haematoma
appendicitis, peritonitis, adnexal torsion, tubo-
ovarian disease, infected haematoma and
Postpartum ovarian vein thrombosis (POVT) pyelonephritis.
Pyelonephritis • Dougan C, Phillips R, Harley I, Benson G,
Torted ovarian cyst Anbazhagan A. Postpartum ovarian vein
thrombosis. The Obstetrician & Gynaecologist
Tubo-ovarian abscess 2016; DOI: 10.1111/tog.12295
8/30/17 ELBOHOTY 405 8/30/17 ELBOHOTY 406

A 26-year-old woman was seen 7 days after a ventouse delivery for • The recommended management of POVT is a
maternal exhaustion in the second stage with a fever, nausea,
vomiting and abdominal pain. When examined she had a
combination of intravenous antibiotics and
temperature of 38°C, mild abdominal distension, an abdominal mass heparin. The antibiotics should be administered for
on deep palpation on the right adnexum and absent bowel sounds. 7−10 days and the recommendation is to continue
An ultrasound scan of the abdomen and pelvis showed features
consistent with an ovarian vein thrombosis. What treatment should with this until 48 hours after leukocytosis has
this patient be offered? resolved. A combination of piperacillin/tazobactam
or carbapenem plus clindamycin provides a broad
Intravenous antibiotics for 7Þ10 days and fully anticoagulated with
fragmin or unfractionated heparin and then continue with warfarin for 3Þ6
coverage in cases of suspected sepsis. The
months recommended anticoagulation should follow the
Intravenous antibiotics for 7Þ10 days and intravenous heparin standards in haematology as recommended,
followed by warfarin for 3Þ6 months which is 3−6 months.
Intravenous heparin followed by warfarin for 3Þ6 months
• Dougan C, Phillips R, Harley I, Benson G,
Intravenous heparin for 3Þ6 months Anbazhagan A. Postpartum ovarian vein
Intravenous heparin for 4Þ5 days followed by subcutaneous heparin thrombosis. The Obstetrician & Gynaecologist
for 3Þ6 months

2016; DOI: 10.1111/tog.12295.
8/30/17 ELBOHOTY 407 8/30/17 ELBOHOTY 408

102
8/30/17

162 .Confirmation of post hystroscopic • c


sterlisation:
A.it work immidiatly
B. X.ray with out time limit
C. HSG in 3month

8/30/17 ELBOHOTY 409 8/30/17 ELBOHOTY 410

163 .The following enhanced recovery in • a


gyn surgery
A. complex carbohydrate drink before major
surgery
B.can drink up to 4 hrs to prevent
dehydration

8/30/17 ELBOHOTY 411 8/30/17 ELBOHOTY 412

103
8/30/17

167 .Which of the following reduce post • b


operative wound infection
A. sheaving use clipers
B . wash with antiseptic solution
C.bowel preparation

8/30/17 ELBOHOTY 413 8/30/17 ELBOHOTY 414

8/30/17 ELBOHOTY 415 8/30/17 ELBOHOTY 416

104

You might also like